Secrets – Pediatric: Gastroenterology

Secrets – Pediatric: Gastroenterology

CLINICAL ISSUES
1. What is the definition of failure to thrive?
Failure to THRIVE (FTT) is a sign, not a diagnosis or a syndrome. It is a term that describes either weight loss or poor weight gain. In more severe cases linear growth and head circumference can be affected. Some specific FTT growth chart–based definitions for children <2 years of age include (1) weight below the 3rd and 5th percentile for age on more than one occasion, (2) weight declines two or more major percentile lines, (3) weight <80% of the ideal weight for age, and (4) a child below
the 3rd or 5th percentile on the weight-for-length curve.

Jaffe AC: Failure to thrive: current clinical concepts, Pediatr REV 32:100–107, 2011.

2. What is the differential diagnosis of FTT?
The causes of FTT can be divided into the following groups:
• Inadequate nutritional intake: not enough food offered, child not taking in enough, excessive juice, formula dilution
• Malabsorption/loss: gastrointestinal (GI) mucosal disease, pancreatic dysfunction, cholestatic liver disease, persistent vomiting
• Increased metabolic demand: congenital heart disease, chronic lung disease, chronic renal failure, acidosis, congenital or chronic infections, chronic systemic disease, genetic syndromes.

Jaffe AC: Failure to thrive: current clinical concepts, Pediatr REV 32:100–107, 2011.

3. How is FTT evaluated?
The most important aspect of the evaluation for FTT is the history and physical examination. History should address feeding, stooling, developmental, psychosocial, and family history. The examination should also focus on any findings that suggest a malformation. Laboratory testing is rarely useful,
but may include a complete blood count (CBC) basic metabolic profile (BMP), urinalysis, urine culture, and lead level.
4. What features of history or on physical examination suggest a medical condition leading to FTT?
History: Recurrent vomiting, chronic diarrhea, frequent infections, and failure to gain weight despite adequate caloric intake
Exam: Dysmorphic features, cardiac exam abnormalities, organomegaly or lymphadenopathy
5. How is the diagnosis of “pinworms” made?
Direct visualization of larger adult worms in the perianal region of a child can sometimes be successful, with the best examination time 2 to 3 hours after the child is asleep. Additionally, transparent adhesive tape can be applied to the perianal region to collect eggs; the tape can be examined under low-power microscopy (Fig. 7-1). These specimens are best obtained in the morning. Because few pinworm ova are present in stool, examination of stool specimens for ova and parasites (for pinworms) is not recommended.

American Academy of Pediatrics: Pinworm infection. In Pickering LK, editor: 2012 Red Book: Report of the Committee on Infectious Diseases, ed 29. Elk Grove Park, IL, 2012, American Academy of Pediatrics, pp 566–567.

220

Figure 7-1. Pinworm eggs as collected on adhesive tape. (From the Public Health Image Library, Centers for Disease Control and PREVENTION: http://phil.CDC.GOV.)

6. What characterizes functional abdominal pain in children?
Previously called recurrent abdominal pain, this entity is common in pediatric practice and refers to children without evidence of inflammatory, anatomic, infectious, allergic, metabolic, or neoplastic processes that explain the symptoms. The cause is likely multifactorial, including abnormalities in the enteric nervous system with possible visceral hyperalgesia or a decreased threshold for pain
in response to changes in intraluminal pressure secondary to physiologic stimuli. This combination of biopsychosocial mechanisms (physiologic, psychological, and behavioral) results in a broad range of management approaches.
7. In children with abdominal pain, what historical features suggest a possible organic or serious cause?
• Involuntary weight loss
• Deceleration of linear growth
• GI blood loss
• Significant vomiting (e.g., bilious emesis, hematemesis, protracted vomiting, cyclical vomiting, pattern concerning to physician)
• Chronic severe diarrhea
• Alarm signs on abdominal examination (right upper or lower quadrant tenderness, localized fullness or mass effect, peritoneal signs, hepatomegaly, splenomegaly, costovertebral angle tenderness)
• Unexplained fevers
• Family history of inflammatory bowel disease or other significant GI illnesses
• Presentation of symptoms before 4 years or after 15 years of age

Rasquin A, DiLorenzo C, Forbes D, et al: Childhood functional gastrointestinal disorders: child/adolescent,
Gastroenterology 130:1527–1537, 2006.
Di Lorenzo C, Colletti RB, Lehmann HP, et al: Chronic abdominal pain in children: a clinical report of the American Academy of Pediatrics and North American Society for Pediatric Gastroenterology, Hepatology, and Nutrition,
J Pediatr Gastroenterol Nutr 40:245–248, 2005.

8. What treatments are used for functional abdominal pain in children?
• Dietary: Low-lactose diets, dietary fiber, low-fructose diets, probiotics
• Pharmacologic: Antidepressants, antispasmodics, prokinetic agents, H2-receptor antagonists, leukotriene-receptor antagonists
• Psychological: Cognitive behavioral therapy, family intervention, relaxation and distraction techniques
• Complementary and alternative medicine: Herbal medicine, peppermint oil, biofeedback, hypnotherapy, massage therapy, acupuncture

Whitfield KL, Shulman RJ: Treatment options for functional gastrointestinal disorders, Pediatr Ann 38:288–294, 2009. Banez GA: Chronic abdominal pain in children: what to do following the medical evaluation, Curr Opin Pediatr 20:571–575, 2008.

9. What is intractable singultus?
Persistent hiccups. Hiccups result from involuntary, spasmodic contracture of the diaphragm accompanied by a sudden closure of the glottis. Persistent hiccups can be a diagnostic and therapeutic challenge with a broad differential diagnosis including some central nervous system (CNS) possibilities, such as seizures and tumors.

Chang FY, Lu CL: Hiccup: mystery, nature and treatment, J Neurogastroenterol Motil 18:123–130, 2012.

10. What is the most commonly ingested foreign body?
Coins account for more than 20,000 visits yearly to emergency departments in the United States. Symptomatic patients are more likely to have the coin lodged in the esophagus, although a significant portion of these patients may be asymptomatic. Coins lodged in the esophagus should be removed endoscopically within 24 hours because of the risk for ulceration and perforation.
11. Which is potentially more dangerous after ingestion: a penny made in 1977 or one made in 1987?
The penny from 1987. In 1982, the composition of pennies changed. Coins minted after that date have higher concentrations of zinc, which is more corrosive and potentially more harmful after prolonged contact with stomach acid.
12. What is the difference radiographically between a coin in the esophagus and a coin in the trachea?
A coin in the esophagus appears en face in the anteroposterior view (sagittal plane), whereas a coin in the trachea appears en face on the lateral view (coronal plane) (Fig. 7-2). This occurs because the cartilaginous ring of the trachea is open posteriorly, but the opening of the esophagus is widest in the transverse position.

Figure 7-2. Anteroposterior (AP) and lateral chest x-rays of a child show the ingested coin in the proximal esophagus. (From Ginsberg CG, Gostout CJ, Kochman M, Norton ID, editors: Clinical Gastrointestinal Endoscopy, ed 4. Philadelphia, Saunders/ ELSEVIER, 2012, p 231.)

13. Why is ingestion of a button battery more dangerous than ingestion of a coin? Button batteries, like coins, often become lodged in the esophagus. If this happens they are capable of causing significant mucosal burns very quickly (sometimes in a matter of hours). In severe cases, batteries can erode through the wall of the esophagus and into surrounding structures including the aorta. This can lead to fatal hemorrhage. For this reason, if there is any suspicion that there is a

button battery stuck in the esophagus, it must be removed emergently. Button batteries that reach the stomach do not pose as much risk but should be followed to ensure passage and removed
if they do not pass out of the stomach within 2 to 3 days. It is also always important to determine the type of button battery ingested.

14. Which is more dangerous, 2 magnets that are swallowed together or 2 magnets that are swallowed separately?
Although swallowed magnets in any form always pose a significant risk, magnets that are swallowed separately pose greater risk. Magnets swallowed together usually attract (stick together) while magnets swallowed separately have the potential to migrate down the bowel separately and subsequently stick to each other across loops of bowel, leading to perforation.

Hussain SZ, Bousvaros A, et al: Management of ingested magnets in children, J Pediatr Gastroenterol Nutr
55:239–242, 2012.

15. What are the indications for emergent foreign body removal?
In general, any object(s) swallowed that may be “stuck in the esophagus” based on symptoms (chest pain, odynophagia, dysphagia, epigastric pain, drooling etc.), including meat impactions, should be removed emergently. Most sharp objects in the stomach should be emergently removed. Objects larger than 2.5 cm wide and 5 cm long are unlikely to pass through the pylorus and should be removed. Patients who report any symptoms (vomiting, pain, fever, dysphagia) need emergent evaluation even if the object is in the stomach or intestine.

16. What is the grim news about Rapunzel syndrome?
Rapunzel syndrome, which results from trichotillomania, is a trichobezoar (a bezoar formed from hair) that can form a cast outline of the stomach and small intestine and over time may extend into the small bowel. Surgical removal is typically the only therapeutic option for removing large trichobezoars such as those seen in the Rapunzel syndrome.

Gonuguntla V, Joshi D-D: Rapunzel syndrome: a comprehensive review of an unusual case of trichobezoar,
Clin Med Res 7:99–102, 2009.

17. What is the most common clinical presentation of juvenile polyps in children? Painless, rectal bleeding. Up to one-third of patients can have chronic blood loss with microcytic anemia. The peak prevalence in children is between 1 and 7 years of age. The polyps are most commonly found in the rectum. Large polyps can be the lead point for intussusception.

18. What are the types of colonic polyps?
• Malignant: Polyps with cells that have lost their normal differentiation (including adenomas, some of which have ability to become cancerous)
• Hamartomatous: Benign focal malformations composed of tissue elements normally found at that site but growing in a disorganized manner
• Hyperplastic: A serrated polyp without malignant potential
• Inflammatory: Polyps associated with inflammatory bowel diseases

19. Why is it important to confirm a diagnosis of juvenile polyposis?
JUVENILE polyposis syndrome is defined by 5 or more polyps in the colon/rectum and 1 or more affected family members. This disorder is inherited in an autosomal dominant fashion. It is common (up to 12%) in patients with symptomatic polyps, especially with right colonic polyps, anemia, and adenomas rather than hamartomas. The importance of establishing a diagnosis of a polyposis syndrome is that some syndromes (e.g., Peutz-Jeghers and juvenile polyposis coli) are associated with a risk for developing adenocarcinoma, with an incidence as high as 30% in as

Figure 7-3. Macular pigmented lesions in patient with Peutz-Jeghers syndrome. (From Kliegman RM, Stanton BF, St. Geme JW, et al, editors: Nelson Textbook of Pediatrics, ed 19. Philadelphia, 2011, Saunders, p 1362.e5.)

few as 10 years after diagnosis. Another feature of Peutz-Jeghers syndrome is the presence of characteristic small, dark-colored spots (melanosis) on the lips, inside the mouth, and near the eyes and nostrils (Fig. 7-3).

Brosens LAA, Langeveld D, et al: Juvenile polyposis syndrome, World J Gastroenterol 17:4839–4844, 2011.

20. How is ascites diagnosed by physical examination?
Severe ascites is commonly diagnosed by observation of the child in a supine and then an upright position. Bulging flanks, umbilical protrusion, and scrotal edema (in males) are generally evident. Three main techniques are used when the diagnosis is not obvious:
• Fluid wave: This sign can be elicited in a cooperative patient by tapping sharply on one flank while receiving the wave with the other hand. The transmission of the wave through fatty tissue should be blocked by a hand placed on the center of the abdomen.
• Shifting dullness: With the patient supine, percussion of the abdomen will demonstrate a central area of tympany at the top that is surrounded by flank percussion dullness. This dullness shifts when the patient moves laterally or stands up.
• “Puddle sign”: A cooperative and mobile patient may be examined in the knee-chest position. The pool of ascites is tapped while you listen for a sloshing sound or change in sound transmission with the stethoscope.
Small amounts of ascites can be extremely difficult to detect with physical examination in children. Although ascites can be demonstrated on radiographs, the most sensitive and specific test is an abdominal-pelvic ultrasound, which can detect as little as 150 mL of ascitic fluid.

CONSTIPATION
21. What constitutes constipation in childhood?
Constipation is defined as a delay or difficulty in defecation, present for 2 or more weeks and sufficient to cause distress in the patient. Normal stool frequency varies from several times a day to three stools per week. In children, constipation should be considered when the normal stooling pattern becomes more infrequent, when stools become hard or are difficult to expel, or when the child exhibits withholding patterns or behavioral changes toward moving his or her bowels. Soiling (encopresis) can be a sign of constipation.

Auth MKH, Vora R, Farrelly P, et al: Childhood constipation, BMJ 345:e7609, 2012.

22. What features suggest an organic etiology for constipation?
• History of weight loss or inadequate weight gain
• Lumbosacral nevi or sinus
• Multiple café-au-lait spots
• Abnormal neurologic examination (decreased tone, strength; abnormal reflexes)
• Anal abnormalities (anteriorly displaced, patulous, or tight)

• Gross or occult blood in stool
• Abdominal distention with or without vomiting

Croffie JM, Fitzgerald JF: Constipation and irritable bowel syndrome. In Liacouras CA, Piccoli DA, editors: Pediatric Gastroenterology: The Requisites in Pediatrics. Philadelphia, 2008, Mosby Elsevier, p 33.

23. What is the most important component of the physical examination when evaluating constipation?
The rectal examination. The presence of large amounts of stool in the rectal vault almost always indicates functional constipation. Lack of stool in the rectal vault could indicate recent evacuation; if expulsion of stool occurs after removal of the examining finger, Hirschsprung disease should be considered. Failure to perform a rectal examination is a common omission during the evaluation of children, and impaction in chronic constipation often goes undetected.

Safder S: Digital rectal examination and the primary care physicians: a lost art? Clin Pediatr 45:411–414, 2006. Gold DM, Levine J, Weinstein TA, et al: Frequency of digital rectal examination in children with chronic constipation, Arch Pediatr Adolesc Med 153:377–379, 1999.

24. What are some common triggers of constipation in healthy infants and children?
• Introduction of solid foods or cow milk: Diet may be low in fiber and not provide adequate fluid intake.
• Inadequate toilet training: Toddlers may not respond appropriately to the need to defecate or may not have adequate foot support needed for effective evacuation of stool if using an adult-sized toilet. If passage of stool is painful, toddlers can begin to withhold stool. If stool is not made softer by increasing fiber and/or fluids in the diet or by stool softeners, this pattern can continue.
• School entry: Children may be reluctant to use the toilet at school, leading to a pattern of stool withholding, painful stools, and constipation.

Borowitz SM, Cox DJ, Tam A, et al: Precipitants of constipation during early childhood, J Am Board Fam Pract
16:213–218, 2013.

KEY POINTS: CONSTIPATION
1. Ninety-nine percent of full-term infants pass stool less than 24 hours after birth. Failure to pass stool within the first 48 hours of life should be considered pathologic until proved otherwise.
2. The rectal examination is a common omission among patients undergoing an evaluation for constipation. Tone, the amount of stool, and the size of the rectal vault should be assessed.
3. Fecal soiling is almost always associated with severe functional constipation and not Hirschsprung disease.
4. Treatment of functional constipation is multimodal and includes medications.
5. Organic causes are suggested by weight loss, lumbosacral nevi, anal abnormalities, blood in stool, and abdominal distention.

25. Which clinical features differentiate chronic retentive constipation from Hirschsprung disease?
See Table 7-1.

26. How is Hirschsprung disease diagnosed?
Hirschsprung disease results from the failure of normal migration of ganglion cell precursors to
their location in the GI tract during gestation. The diagnosis can be made by obtaining an unprepped barium enema, which will demonstrate a change in the caliber of the large intestine at the site where normal bowel meets aganglionic bowel (transition zone) (Fig. 7-4). An unprepped barium enema is required because the use of cleansing enemas can dilate the abnormal portion of the

Table 7-1. Clinical Distinctions Between Chronic Retentive Constipation and Hirschsprung Disease

CLINICAL FEATURE FUNCTIONAL CONSTIPATION HIRSCHSPRUNG DISEASE
Age of onset >1 yr <1 yr
Passage of meconium Within 24 hr Meconium passes after 24 hr
Abdominal pain Frequent, colicky Rare
Stool size Large Small, ribbonlike
Stool withholding behavior Present Absent
Encopresis (soiling) Present Very rare
Rectum Filled with stool Empty
Rectal examination Stool in rectum Explosive passage of stool
Growth Normal Poor

Figure 7-4. Contrast enema in a newborn with Hirschsprung disease. Note the transition zone (arrow) as the more dilated proximal colon tapers to a more narrow distal colon at the rectosigmoid junction. (From Liacouras CA, Piccoli DA: Pediatric Gastroenterology: The Requisites in Pediatrics, Philadelphia, 2005, ELSEVIER Mosby, p 1170.)

colon and remove some of the distal impaction, thereby resulting in a false-negative result. After the study, the retention of barium for 24 or more hours is suggestive of Hirschsprung disease
or a significant motility disorder. This study is less reliable in a child younger than 6 months. Rectal suction biopsies or full-thickness surgical biopsies will confirm the absence of ganglion cells. Anal manometry is less reliable in children; in small infants, it requires specialized equipment.
27. What is the most common cause of encopresis?
Encopresis, or fecal soiling, may be defined as the involuntary passage of fecal material in an otherwise healthy and normal child. The most common cause is functional constipation with overflow incontinence. Children with encopresis typically sense no urge to defecate. Fecal soiling is almost always associated with severe functional constipation.
28. How should children with chronic constipation and encopresis be managed?
• The rectosigmoid colon should be aggressively cleansed of fecal material. Manual disimpaction is sometimes required. Multiple enemas over multiple days are commonly needed. Adult enemas should be used in children who are older than 3 years.

• Medications that act as an osmotic laxative by drawing fluid into the intestine to promote the passage of soft stools include polyethylene glycol powder and lactulose (a nonabsorbable sugar). Other osmotic agents, such as sorbitol and magnesium citrate, can be considered. For cases of long-standing functional constipation, osmotic laxatives should be continued for a minimum of several months while the dilated rectum returns to normal size.
• An oral lubricant, such as mineral oil, can help promote the continued passage of stool but
can contribute to accidental soiling. In difficult cases, stimulant medications, such as bisacodyl or senna, can be substituted for short-term use.
• It is extremely important to educate patients and parents about the mechanics of the disorder. A high-fiber diet, possible limitation of dairy and complex carbohydrates, defined periods of toilet sitting (2 to 3 times daily for 10 minutes after meals), and a behavior modification system that rewards normal bowel movements are essential for eventual success. Integrative approaches of biofeedback, relaxation strategies, and mental imagery have been used for children who have severe “defecation anxiety.” A goal is one to two soft bowel movements a day. Relapses are common.

Har AF, Croffie JM: Encopresis, Pediatr REV 31:368–374, 2010.

DIARRHEA AND MALABSORPTION
29. What time frame distinguishes acute and chronic diarrhea? Diarrhea is the frequent (>3 times per day) evacuation of liquid feces. Acute diarrhea is often self-limiting and lasts for a few days. Diarrhea is considered chronic when lasting >3 weeks.
30. What is the most common cause worldwide of epidemic diarrhea?
Norovirus. These single-stranded RNA viruses are believed to be responsible for at least 50% of all gastroenteritis outbreaks worldwide and a major cause of foodborne illness. With the widespread use of rotavirus vaccination, norovirus has become the most common cause of medically attended acute gastroenteritis in children <5 years in the United States.

Payne DC, Vinjé J, Szilagyi PG, et al: Norovirus and medically-attended gastroenteritis in the U.S. children, N Engl J Med
368:1121–1130, 2013.
Hall AJ, Vinjé J, Lopman B, et al: Updated Norovirus outbreak management and disease prevention guidelines, MMWR 60 (RR03):1–15, 2011.

31. What are other common causes of acute diarrhea?
• Viral (others include rotavirus, enterovirus)
• Bacterial (e.g., Escherichia coli, Shigella, Salmonella, Yersinia, Campylobacter, Clostridium difficile)
• Protozoal
• Allergic
• Medication side effect (e.g., antibiotic usage)
• Extra-intestinal infections (e.g., respiratory, urinary, sepsis)
32. Which historical questions are key when seeking the cause of diarrhea?
• Recent medications, especially antibiotics
• History of immunosuppression (e.g., recurrent major infections, history of malnutrition, acquired immunodeficiency syndrome, immunosuppressive medications)
• Illnesses in other family members or close contacts
• Travel outside of the United States
• Travel to rural or seacoast areas (i.e., involving the consumption of untreated water, raw milk, or raw shellfish)
• Attendance in day care
• Recent foods particularly focus on juice and fructose consumption
• Presence of family pets
• Food preparation and water source

Thielman NM, Guerrant RL: Acute infectious diarrhea, N Engl J Med 350:38–47, 2004.

33. In what settings can diarrhea be a severe, life-threatening illness?
Severe diarrhea of any cause can lead to dehydration, which can cause significant morbidity and mortality. However, diarrhea can be a sign of a serious associated illness, which in itself can be life-threatening:
• Intussusception
• Salmonella gastroenteritis (neonatal or compromised host)
• Hemolytic-uremic syndrome
• Hirschsprung disease (with toxic megacolon)
• Pseudomembranous colitis (classically due to C. difficile)
• Inflammatory bowel disease (with toxic megacolon)

Fleisher GR: Diarrhea. In Fleisher GR, Ludwig S, editors: Textbook of Pediatric Emergency Medicine, ed 6. Philadelphia, 2010, Wolters Kluwer, p 213.

34. Why is true diarrhea during the first few days of life especially concerning?
In addition to the greater potential for dehydration in a newborn, diarrhea in this age group is more commonly associated with major congenital intestinal defects involving electrolyte transport (e.g., congenital sodium- or chloride-losing diarrhea), carbohydrate absorption (e.g., congenital lactase deficiency), immune-mediated defects (e.g., autoimmune enteropathy), or those characterized by villous blunting (e.g., microvillus inclusion disease). Although viral enteritis can occur in the nursery, any newborn with true diarrhea warrants thorough evaluation and possible referral to a tertiary center.

Sherman PM, Mitchell DJ, Cutz E: Neonatal enteropathies: defining causes of protracted diarrhea in infancy,
J Pediatr Gastroenterol Nutr 30:16–26, 2004.

35. What are the most useful stool tests for diagnosing fat malabsorption? Measurement of 72-hour fecal fat is the gold standard test for fat malabsorption. The patient must ingest a high-fat diet for 3 to 5 days (100 g daily for adults), and all stool is collected for the final 72 hours. A complete and accurate dietary history should be obtained concomitantly so the coefficient of fat absorption can be calculated. Steatorrhea is present if more than 7% of dietary fat is malabsorbed. In normal infants, up to 15% of fat can be malabsorbed. Other tests include
Sudan staining of stool for fat globules (a qualitative test that, if positive, indicates gross steatorrhea), the steatocrit, and monitoring absorbed lipids after a standardized meal.

36. What stool test is most useful for helping diagnose GI protein loss?
Fecal α1-antitrypsin measurement is the most useful stool marker of protein malabsorption.
It is important to concomitantly measure serum α1-antitrypsin to ensure that the patient does not have α1-antitrypsin deficiency, which could result in a false-negative stool study.

37. How do patterns of secretory or enterotoxigenic and inflammatory diarrhea vary? Secretory or enterotoxigenic disease is characterized by watery diarrhea and the absence of fecal leukocytes. Inflammatory disease is characterized by dysentery (i.e., symptoms and bloody stools), as well as fecal leukocytes and red blood cells.

38. What is the primary pathophysiologic difference between secretory and osmotic diarrhea?
In osmotic diarrhea, undigested nutrients increase the osmotic load in the distal small intestine and the colon leading to decreased water absorption. In secretory diarrhea, a noxious agent causes the intestinal epithelium to secrete excessive water and electrolytes into the lumen.

39. How can osmotic diarrhea be distinguished from secretory diarrhea?
In true osmotic diarrhea, symptoms should cease when the patient is made NPO. In addition, a
fecal osmotic gap can be calculated. In osmotic diarrhea, the fecal electrolyte content becomes lower than the serum. Stool electrolytes should be collected and compared with a normal serum osmolality, 290 mOsm/kg. The fecal osmotic gap is [290 2(Na+ K)].
See Table 7-2.

Table 7-2. Osmotic Diarrhea Versus Secretory Diarrhea
STOOLS OSMOTIC DIARRHEA SECRETORY DIARRHEA
Electrolytes Na+ <70 mmol/L Na+ >70 mmol/L
Cl— <25 mEq/L Cl— >40 mEq/L
Osmotic gap*
>135 mOsm <50 mOsm
pH <5.6 >6.0
Response to fasting Improvement None
*The osmotic gap is the osmolality of the fecal fluid minus the sum of the concentrations of the fecal electrolytes. From Guarino A, DeMarco G: Persistent diarrhea In Walker WA, Goulet O, Kleinman RE, et al, editors: Pediatric Gastrointestinal Disease, 4th ed. Hamilton, Ontario, 2004, BC Decker, pp 180–193.

40. How should children with secretory diarrhea be managed?
After the child is taken off feeds, a vigorous attempt must be initiated to maintain fluid and electrolyte balance. If this is successful, the child should be evaluated for proximal small bowel damage, enteric pathogens, and a baseline malabsorption workup. If abnormalities of the mucosal integrity are suspected, a small bowel biopsy is performed; if the findings are significantly abnormal, the patient may be given parenteral alimentation and gradual refeeding. Electron microscopy may reveal congenital abnormalities of the microvillus membrane and the brush border. Hormonal causes of secretory diarrhea (e.g., a VIPoma, hypergastrinoma, or carcinoid syndrome) must be considered if initial studies are negative.

41. What rare tumors can cause true secretory diarrhea?
• Gastrinoma: Children present with typical ulcer pain, hematemesis, vomiting, and melena. High acid output into the proximal small bowel leads to precipitation of bile salts and steatorrhea.
• VIPoma: Children present with profuse watery diarrhea with marked fecal losses (20 to 50 mL/kg/day) due to high levels of vasoactive intestinal peptide (VIP).

42. What features characterize “toddler diarrhea”?
Toddler diarrhea, which is also known as chronic nonspecific diarrhea, is a clinical entity of unclear etiology that occurs in infants between 6 and 40 months of age, often following a distinct identifiable enteritis and treatment with an antibiotic. Loose, nonbloody stools (at least two per day
but usually more) occur without associated symptoms of fever, pain, or growth failure. Malabsorption is not a key feature.
Multiple causes may be present: overconsumption of fruit juices, relative intestinal hypermotility, increased secretion of bile acids and sodium, and intestinal prostaglandin abnormalities.
The diagnosis is one of exclusion, and toddlers should be evaluated for disaccharide intolerance, protein hypersensitivity, parasitic infestation, and inflammatory bowel disease. Treatment consists of reassurance, careful growth assessment, and psyllium bulking agents (as initial therapy). Other agents used with success have been cholestyramine and metronidazole.

43. What is primary lactose intolerance?
In more than 50% of the population, beginning at the age of 5 years, lactase levels decline progressively after having been normal in infancy. These levels decline at different rates for different people depending on their genetics. Most adults with primary lactose intolerance have lactase levels of about 10% of those seen during infancy. Symptoms of lactose intolerance (e.g., bloating, nausea, cramps, diarrhea after dairy foods) may develop if excessive lactose loads are ingested.

44. How does late-onset lactase deficiency vary by ethnicity?
See Table 7-3.

Table 7-3. Approximate Percentage of Low Lactase Activity by Ethnic Group
UNITED STATES WORLDWIDE
White 20% Dutch 0%
Hispanic 50% French 32%
Black 75% Filipino 55%
Native American 90% Vietnamese 100%

45. What conditions produce secondary lactose intolerance?
Any disorder that alters the mucosa of the proximal small intestine may result in secondary lactose intolerance. For this reason, the lactose tolerance test is commonly used as a screening test for intestinal integrity, although this has the disadvantage of concomitantly identifying all primary lactose malabsorbers. Although a combination of factors is present in many disease processes, secondary lactose intolerance can be organized into lesions of the microsurface, total surface, transit time, and site of bacterial colonization in the small bowel.
Microvillus and brush border:
• Post-enteritis
• Bacterial overgrowth
• Inflammatory lesions (Crohn disease)
Level of the villus:
• Celiac disease
• Allergic enteropathy
• Eosinophilic gastroenteropathy
Bulk intestinal surface area:
• Short bowel syndrome
Altered transit with early lactose entry into colon:
• Hyperthyroidism
• Dumping syndromes
• Enteroenteral fistulas
46. How is lactose intolerance diagnosed?
The most common noninvasive method of diagnosing lactose intolerance is a breath hydrogen test. The fasted patient is fed 2 g/kg (up to 25 g) of lactose, and end-expired air is collected every 15 minutes for the next 2 to 3 hours for the purpose of measuring hydrogen concentration. Fermentation of carbohydrate by bacteria in the colon results in hydrogen expiration after lactose ingestion. A peak hydrogen level of 20 parts per million above the baseline after about 60 minutes in concert with a symptomatic response is considered a positive test. Because of the need for colonic bacteria to ferment carbohydrate and produce hydrogen gas, it is important that the patient not receive antibiotics immediately before the test.
Direct measurement of lactase levels, as well as the other disaccharidases, can be obtained by biopsy of the duodenum or jejunum during upper endoscopy.

Heyman MB: Lactose intolerance in infants, children and adolescents, Pediatrics 118:1279–1286, 2006.

47. What is the role of stool elastase measurement? Measurement of fecal pancreatic elastase is a screen for pancreatic insufficiency, which can be a cause of fat malabsorption (e.g., cystic fibrosis). A decreased measurement of pancreatic elastase is associated with pancreatic insufficiency, although values can be falsely decreased when the sample is obtained from diarrheal specimen.
48. What three individual clinical features are the most accurate for predicting 5% dehydration?
• Abnormal capillary refill
• Abnormal skin turgor
• Abnormal respiratory pattern

Steiner MJ, DeWalt DA, Byerley JS: Is this child dehydrated? JAMA 291:2746–2754, 2004.

49. How accurate are urine specific gravity and blood urea nitrogen (BUN) measurements as means of assessing dehydration in children?
Notoriously unreliable. Although a high urine specific gravity is commonly thought to be associated with dehydration, a prospective study of 75 dehydrated children found the correlation was poor. BUN does not begin to rise until the glomerular filtration rate falls to about one-half of normal; it then
rises by about 1% each hour, and it may rise even less in a fasting child with disease. In a prospective study, Bonadio and colleagues found that 80% of patients judged to be 5% to 10% dehydrated by common physical findings may have a normal BUN.

Steiner MJ, Nager AL, Wang VJ: Urine specific gravity and other urinary indices: inaccurate tests for dehydration,
Pediatr Emerg Care 23:298–303, 2007.
Bonadio WA, Hennes HH, Machi J, Madagame E: Efficacy of measuring BUN in assessing children with dehydration due to gastroenteritis, Ann Emerg Med 18:755–757, 1989.

50. How do various oral rehydration solutions differ in composition from other liquids that are commonly used for rehydration?
Many home remedies are either very deficient or very excessive in electrolytes or sugar. Chicken broth has no carbohydrates and very high sodium. Sodas (such as colas) can have up to 8 the recommended sugar content with negligible sodium and potassium. Apple has very high sugar content and very high osmolality and negligible sodium. Tea has neither carbohydrate nor sodium. Commercially available oral rehydration solutions incorporate carbohydrates (25 to 50 g/L), sodium (45 to 90 mEq/L), and potassium (20 to 25 mEq/L) to maximize coupled transport.

51. How can the World Health Organization (WHO) oral electrolyte (rehydration) solution be duplicated?
The WHO solution is 2% glucose, 20 mEq K+/L, 90 mEq Na+/L, 80 mEq Cl—/L, and 30 mEq bicarbonate/L. This solution is approximated by adding ¾ tsp of salt, 1 tsp of baking soda, 1 cup of orange juice (for KCl), and 8 tsp of sugar to 1 L of water.

52. What traditional approaches to feeding during diarrhea are no longer recommended and should be avoided?
• Switching to lactose-free formula: This is usually unnecessary because, for most infants, clinical trials have not shown an advantage. Certain infants with severe malnutrition and dehydration may benefit from lactose-free formula.
• Diluted formula: Half- or quarter-strength formula has been shown in clinical trials to be unnecessary and associated with prolonged symptoms and delays in nutritional recovery.
• Clear liquids: Foods high in simple sugars (e.g., carbonated soft drinks, juice drinks, gelatin desserts) should be avoided because the high osmotic load might worsen diarrhea.
• Avoid fatty foods: Fat may have a beneficial effect of reducing intestinal motility.
• BRAT diet: The bananas, rice, applesauce, and toast diet is unnecessarily restrictive and can provide suboptimal nutrition.
• Avoid food for at least 24 hours: Early feeding decreases the intestinal permeability caused by infection, reduces illness duration, and improves nutritional outcome.

King CK, Glass R, Bresee JS, Duggan C, Centers for Disease Control and Prevention: Managing acute gastroenteritis among children: oral rehydration, maintenance, and nutritional therapy, MMWR Recomm Rep 52:1–16, 2003.

53. What is the role of antiemetic agents in children with gastroenteritis?
Published guidelines have not yet formally recommended the use of antiemetic medications, particularly domperidone, metoclopramide, prochlorperazine, and promethazine, because of concerns of increased emergency department (ED) revisits rates of misdiagnosis, and health care costs. Oral ondansetron, a centrally acting 5-hydroxytryptamine antagonist, has been found to be useful in decreasing the risk for persistent vomiting, lessening the need for intravenous therapy in ED settings and reducing the likelihood of hospitalization.

Freedman SB, Hall M, Shah SS, et al: Impact of increasing ondansetron use on clinical outcomes in children with gastroenteritis, JAMA Pediatr 168:321–329, 2014.

54. What are non-antimicrobial drug therapies for diarrhea?
In older children, adolescents, and adults, the following categories are used. Pediatric data are limited, and these medications are not typically approved or recommended for children
<3 years of age.
• Antimotility agents (loperamide [Imodium], diphenoxylate and atropine [Lomotil], tincture of
opium [Paregoric]): These can cause drowsiness, ileus, and nausea and potentiate the effects of certain bacterial enteritides (e.g., Shigella, Salmonella) or accelerate the course of antibiotic- associated colitis.
• Antisecretory drugs (bismuth subsalicylate [Pepto-Bismol]): These involve the potential for salicylate overdose.
• Adsorbents (attapulgite, kaolin-pectin [Donnagel, Kaopectate]): These can cause abdominal fullness and interfere with other medications.
55. What is the role of probiotic organisms in the treatment of antibiotic-associated diarrhea?
Probiotics (which are the opposite of antibiotics) are living organisms that are believed to cause health benefits by replenishing some of the more than 500 species of intestinal bacteria that antibiotics can suppress and by inhibiting the growth of more pathogenic flora. Among children receiving broad-spectrum antibiotics, about 20% to 40% are likely to experience some degree of diarrhea. Lactobacillus GG, Bifidobacterium bifidum, and Streptococcus thermophilus have been shown to limit antibiotic-associated diarrhea in children.

Applegate JA, Fischer Walker CL, Ambikapathi R, et al: Systematic review of probiotics for the treatment of community-acquired acute diarrhea in children, BMC Public Health 13:S3–S16, 2013.

KEY POINTS: DIARRHEA AND MALABSORPTION
1. History is crucial to diagnosis and should include recent medications, ill family contacts, travel, attendance at school or day care, pets, and water sources.
2. Three keys to the assessment of dehydration are (1) capillary refill, (2) skin turgor, and (3) respiratory pattern.
3. Salmonella species infection is more concerning among infants who are younger than 1 year because of the increased risk for dissemination (e.g., bacteremia, meningitis).
4. Toddler diarrhea is a common cause of chronic diarrhea in children between the ages of 6 and 40 months.
5. Celiac disease (a sensitivity to gluten) is common (up to 1% of the general population) and can present with subtle and varied symptoms.
6. Allergic or nonspecific colitis is the most common cause of bloody diarrhea in infants younger than 1 year.

56. Why is Salmonella enteritis so concerning in a child who is younger than 12 months?
In older children with Salmonella gastroenteritis, secondary bacteremia and dissemination of disease rarely occur. In infants, however, 5% to 40% may have positive blood cultures for
Salmonella, and in 10% of these cases, Salmonella can cause meningitis, osteomyelitis, pericarditis, and pyelonephritis. Thus, in infants who are younger than 1 year, outpatient management of diarrhea assumes even greater significance, particularly if Salmonella is suspected.
57. What are the clinical manifestations of typhoid fever? Typhoid fever is caused by Salmonella species typhi and paratyphi. It is characterized by fever, abdominal pain, nausea, decreased appetite, and constipation over the first week. The fever is sometimes paradoxically associated with bradycardia (Faget sign or sphygmothermic dissociation). Leukopenia
is common. Diarrhea begins after approximately a week. If untreated it can last for 2 to 3 weeks and cause significant weight loss and melena. Treatment of typhoid fever is necessary only in patients with sepsis or bacteremia with signs of systemic toxicity or a metastatic focus, which can include otitis, endocarditis, cholecystitis, or encephalitis.

58. Who was Typhoid Mary?
In 1907, a JAMA article traced a series of outbreaks of Salmonella-triggered typhoid fever in 7 families over a 7-year period to the same cook, Mary Mallon, who had been employed by each family during that period. She was subsequently found to be a carrier of Salmonella, the first asymptomatic typhoid carrier identified in the United States. Much of the remainder of her life was spent in an imposed quarantine.

Marineli F, Tsoucalas G, Androutsos G: Mary Mallon (1869-1938) and the history of typhoid fever, Ann Gastroenterol
26:132–134, 2013.

59. What is the most common cause of travelers’ diarrhea? Enterotoxigenic E. coli is clearly the most commonly identified cause of traveler’s diarrhea. Depending on the location, however, other bacteria (such as Campylobacter in Southeast Asia), viruses (norovirus, rotavirus), or parasites (Giardia, Cryptosporidium) can be present.
60. How can travelers’ diarrhea be prevented?
• Avoidance: In high-risk areas of developing countries, avoid previously peeled raw fruits and vegetables and any foods or beverages or ice cubes prepared with tap water.
• Bismuth subsalicylate: Prophylactic bismuth subsalicylate (Pepto-Bismol) has been shown to minimize diarrheal illness in up to 75% of adults. Although some authorities recommend its use in children, others argue against it because of the risk for salicylate intoxication. It can interfere with absorption of doxycycline used for malaria prevention.
• Anti-infective drugs: Prophylactic use of antimicrobial agents such as trimethoprim- sulfamethoxazole, azithromycin, neomycin, doxycycline, and fluoroquinolones can decrease the frequency of travelers’ diarrhea in children and adults. However, routine use of antibiotics is not recommended because of potential risks for allergic drug reactions, antibiotic-associated colitis, and the development of resistant organisms.
• Immunization: Although potentially an ideal solution, at present it is not an alternative.

Hill DR, Ryan ET: Management of travelers’ diarrhea, BMJ 337:863–867, 2008.

61. Which bacterial gastroenteritides may benefit from antimicrobial therapy?
See Table 7-4.

Table 7-4. Benefits of Antimicrobial Therapy in Specific Bacterial Gastroenteritides
ENTEROPATHOGEN INDICATION FOR OR EFFECT OF THERAPY
Shigella species Shortens duration of diarrhea Eliminates organisms from feces
Campylobacter jejuni Shortens duration Prevents relapse
Salmonella species Indicated for infants <12 mo
Bacteremia
Metastatic foci (e.g., osteomyelitis)
Enteric fever
Immunocompromise
Escherichia coli
Enteropathogenic Use primarily in infants Intravenous use if invasive disease
Enterotoxigenic (ETEC) Most illnesses brief and self-limited
Enteroinvasive Mimics shigellosis with diarrhea and high fever
Yersinia enterocolitica None for gastroenteritis alone but indicated if suspected septicemia or other localized infection
Clostridium difficile 10-20% relapse rate

62. What strains of E. coli are associated with diarrhea?
• Enterotoxigenic (ETEC): responsible for travelers’ diarrhea
• Enteropathogenic (EPEC): similar mechanism as ETEC; adheres to epithelial cells and releases toxins that induce intestinal secretions and limit absorption; responsible for epidemics in daycare settings and nurseries
• ENTEROINVASIVe (EIEC): invades mucosa and causes bloody diarrhea
• Enterohemorrhagic (EHEC): produces a Shiga-like toxin that is responsible for hemorrhagic colitis; usually associated with contaminated food and undercooked beef; usually a self-limited gastroenteritis
63. What clinical entity has been attributed to EHEC, specifically strain O157:H7? Hemolytic uremic syndrome (HUS), which is the triad of microangiopathic hemolytic anemia, thrombocytopenia, and renal failure.
64. What is the most common cause of antibiotic-associated colitis?
Clostridium difficile (C. difficile). Fever, abdominal pain, and bloody diarrhea begin as early as a few days after starting antibiotics (especially clindamycin, ampicillin, and cephalosporins). Definitive diagnosis is made by sigmoidoscopy, which reveals pseudomembranous plaques or nodules (Fig. 7-5).

Figure 7-5. Flexible sigmoidoscopy showing adherent plaques (arrows) typical of pseudomembranous colitis. (From Barker HC, Haworth CS, Williams D: Clostridium difficile pancolitis in adults with cystic fibrosis, J Cyst Fibros 7(5):444–447, 2008.)

65. How is the diagnosis of C. difficile made?
C. difficile causes diarrhea by producing two diarrheagenic toxins (A and B). Essay immunoassay for the toxins was previously the diagnostic test of choice. However, in 2013, the American College of Gastroenterology recommended that nucleic acid amplification tests, such as PCR assays, which detect toxin-encoding genes should be the standard diagnostic test because of superior sensitivity and specificity.

Surawicz CM, Brandt LJ, Binion DG, et al: Guidelines for diagnosis, treatment, and prevention of Clostridium difficile
infections, Am J Gastroenterol 28:1219–1227, 2013.

66. How common is asymptomatic C. difficile carriage?
Colonization rates in infants can be up to 70%, with percentages decreasing with age. By the second year of life, the rate declines to about 6%, and above age 2 years to 3%, which is the approximate rate in adults. These high colonization rates make the interpretation of positive tests in younger infants problematic. Toxin assays are more indicative of C. difficile–associated disease than culture. However, the toxin may be present without any symptoms, especially in infants, who

typically do not have the toxin receptors necessary for disease. Unless there is evidence of histologic colitis, asymptomatic carriers do not require treatment.

Bryant K, McDonald LC: Clostridium difficile infections in children, Pediatr Infect Dis J 28:145–146, 2009.

67. Why are alcohol-based sanitizers insufficient when examining patients with C. difficile?
Typical alcohol-based hand hygiene products do not kill the spores of C. difficile. In addition to standard contact precautions (which include gloves at all times and gowns for direct contact with the patient or items in the room), handwashing with soap and water is recommended to more effectively remove spores from contaminated hands.

68. What are the three most common presenting symptoms of giardiasis?
• Asymptomatic carrier state
• Chronic malabsorption with steatorrhea and FTT
• Acute gastroenteritis with diarrhea, weight loss, abdominal cramps, abdominal distention, nausea, and vomiting

69. How reliable are the various diagnostic methods for detecting Giardia?
• Single stool examination for trophozoites or cysts: 50% to 75% (Fig. 7-6)
• Three stool examinations (ideally 48 hours apart) for same: 95%
• Single stool examination and stool enzyme-linked immunosorbent assay test for Giardia
antigen: >95%
• Duodenal aspirate or string test: >95%
• Duodenal biopsy (gold standard): Closest to 100%

Figure 7-6. Trichrome stain of stool revealing cystic form of Giardia (in center). (From Liacouras CA, Piccoli DA: Pediatric Gastroenterology: The Requisites in Pediatrics. Philadelphia, 2005, ELSEVIER Mosby, p 7.)

70. What are the potential complications of amebiasis?
The parasite Entamoeba histolytica disseminates from the intestine to the liver in up to 10% of patients and to other organs less commonly.
• Liver abscess
• Pericarditis

• Cerebral abscess
• Empyema

Haque R, Huston CD, Hughes M, et al: Amebiasis, N Engl J Med 348:1565–1573, 2003.

71. What is gluten?
After starch has been extracted from wheat flour, gluten is the residue that remains. This residue is made up of multiple proteins that are distinguished by their solubility and extraction properties.
For example, the alcohol-soluble fraction of wheat gluten is wheat gliadin. It is this protein component that is primarily responsible for the mucosal injury that occurs in the small bowel in patients with celiac disease. The alcohol-soluble components of barley and rye are also toxic.
72. What classic clinical features suggest celiac disease?
Gluten-sensiTIVE enteropathy (celiac disease) is a relatively common cause of severe diarrhea and malabsorption in infants and children. The classic presentation of celiac disease is a 9- to 24-month-old child with FTT, diarrhea, abdominal distention, muscle wasting, and hypotonia. After several months of diarrhea, growth slows; weight typically decreases before height. Often, these children become irritable and depressed and display poor intake and symptoms of carbohydrate malabsorption.
Vomiting is less common. On examination, the growth defect and distention are commonly striking. There may be a generalized lack of subcutaneous fat, with wasting of the buttocks, shoulder girdle, and thighs. Edema, rickets, and clubbing may also be seen. Many patients with celiac disease, however, have a more subtle presentation rather than the classic constellation of symptoms and can present at an older age.

DiSabatino A, Corazza GR: Coeliac disease, Lancet 373:1480–1493, 2009.

73. What are possible nongastrointestinal manifestations of celiac disease?
• Dermatitis herpetiformis
• Iron deficiency anemia (unresponsive to treatment with oral iron supplements)
• Arthritis and arthralgia
• Dental enamel hypoplasia
• Chronic hepatitis
• Osteopenia and osteoporosis
• Pubertal delay
• Short stature
• Hepatitis
• Arthritis

Telega G, Bennet TR, Werlin S: Emerging new clinical patterns in the presentation of celiac disease, Arch Pediatr Adolesc Med 162:164–168, 2008.

74. What is the appropriate screening test for celiac disease?
Anti–tissue transglutaminase (TTG) immunoglobulin A (IgA) and anti–endomysial antibodies (EMA) IgA have been demonstrated to be highly sensitive and specific for celiac disease. Because of low cost, ease of test performance, and reliability, TTG is currently recommended for initial screening of celiac disease. Antigliadin antibodies, previously the most commonly employed screening test, are not as sensitive or specific for celiac disease and are currently not recommended as first-line screening. However, more advanced methods of measurement of antigliadin have shown promise. Antibodies found in patients with celiac disease are IgA antibodies. Selective IgA deficiency
is the most common primary immunodeficiency in Western countries, with a prevalence of 1.5 to 2.5 per 1,000, and is even more common in patients with celiac disease. Therefore, a quantitative IgA level should be included when measuring screening antibodies.

Tran TH: Advances in pediatric celiac disease, Curr Opin Pediatr 26:585–589, 2014.
Guideline for the Diagnosis and Treatment of Celiac Disease in Children: Recommendations of the North American Society of Pediatric Gastroenterology, Hepatology, and Nutrition, J Pediatr Gastroenterol Nutr 40:1–19, 2005.

75. What is the definitive way to diagnose celiac disease?
Definitive diagnosis of celiac disease requires multiple small bowel biopsies VIA endoscopy while the patient is on a gluten-containing diet. Intestinal biopsies obtained on gluten may show a number of abnormalities including villous atrophy, elongated crypts, increased crypt mitoses, increased intraepithelial lymphocytes, plasma cell infiltrate in lamina propria, absence of brush border, and disorganization and flattening of the columnar epithelium (“villous blunting”). These abnormalities should resolve fully with repeat biopsies after a strict gluten-free diet. Recent European guidelines have recommended that the need for confirmatory biopsy can be omitted in children with clear symptoms of celiac disease, with high levels of transglutaminase antibody and with positive HLA typing. Celiac disease is strongly associated with HLA-DQ types 2 and 8.

Husby S, Koletzko S, Korponay-Szabó IR, et al: European Society for Pediatric Gastroenterology, Hepatology, and Nutrition guidelines for the diagnosis of coeliac disease, J Pediatr Gastroenterol Nutr 54:136–160, 2012.

76. What is the mainstay of treatment for celiac disease?
A strict gluten-free diet needs to be followed throughout life, although nearly one in four patients continues to experience gastrointestinal symptoms. Gluten-free diets should be without wheat, barley, and rye. Upon initial diagnosis, most recommend avoiding oats because of contamination, but eventually most patients with celiac can tolerate oats. Good substitutions are rice and corn flour products.

Paarlahti P, Kurppa K, Ukkola A, et al: Predictors of persistent symptoms and reduced quality of life in treated coeliac disease patients: a large cross-sectional study, BMC Gastroenterol 13:75, 2013.
Celiac Disease Foundation: www.celiac.org. Accessed Nov. 24, 2014. Gluten Intolerance Group: www.gluten.net. Accessed on Nov. 24, 2014.

ESOPHAGEAL DISORDERS
77. What is the likely diagnosis for an infant with excessive secretions and choking episodes in whom a nasogastric tube cannot be passed into the stomach?
Esophageal atresia with tracheoesophageal fistula. This congenital anomaly is usually diagnosed during the newborn period, often when a chest radiograph reveals the intended nasogastric tube coiled in the upper esophageal pouch with the stomach distended with air. Treatment is surgical. The possible variations are shown in Figure 7-7.
78. What underlying diagnoses should be considered in a patient who presents with a meat impaction in the esophagus?
• Eosinophilic esophagitis
• Achalasia
• Esophageal stricture, congenital or acquired
• Prior esophageal surgery
Of note, in children with esophageal food impaction, endoscopy and biopsy reveal an underlying pathologic and potentially treatable etiology in the majority of patients.

Hurtado CW, Furuta GT, Kramer RE: Etiology of food impactions in children, J Pediatr Gastroenterol Nutr 52:43–46, 2011.

79. What is the most common condition that might present as a food impaction in an adolescent?
Eosinophilic esophagitis (EoE). Occurring in children and adults, EoE is characterized by multiple symptoms that are suggestive of gastroesophageal reflux (GER), including heartburn, emesis, regurgitation, epigastric pain, and feeding difficulties, which are typically unresponsive to acid suppression therapy. Pathologically, this is characterized by eosinophilic inflammation of the esophagus and is almost always related to food antigens. In adolescents and adults, EoE often presents with symptoms of dysphagia or, occasionally, food impaction.
80. How is EoE diagnosed?
The diagnosis of EoE requires upper endoscopy. According to the most recent EoE guidelines, EoE is a defined clinicopathologic diagnosis that consists of isolated esophageal dysfunction and

A

No EA
C but “H” fistula

EA and
D 2 fistulas

EA and
E proximal fistula

Figure 7-7. A, Esophageal atresia with distal esophageal communication with the tracheobronchial tree (most common type: 80%). B, Esophageal atresia without a distal communication. C, H-type fistulas between otherwise intact trachea and esophagus. D, Esophageal atresia with both proximal and distal communication with the trachea. E, Esophageal atresia with proximal communication. (From Blickman H, editor: The Requisites: Pediatric Radiology, ed 2. Philadelphia, 1998, Mosby, p 93.)

esophageal biopsies with greater than 15 eosinophils per high power field. Other causes of an isolated esophageal eosinophilia must be excluded, specifically GER and proton pump inhibitor responsive esophageal eosinophilia. Patients must be on an adequate dosage proton pump inhibitor for a period of 8 weeks before any biopsy.

Liacouras CA, Furuta GT, et al: Eosinophilic esophagitis: updated consensus recommendations for children and adults,
J Allergy Clin Immunol 128:3–30, 2011.

81. What are common endoscopic findings in EoE?
• Esophageal furrowing and edema
• Esophageal rings or “trachealization” (Fig. 7-8)
• White plaques
• Esophageal strictures
• Mucosal tearing

Figure 7-8. “Trachealization” or “felinization” of the midesophagus of a patient with EoE. The terms arise from the ringed appearance of the esophagus that cause it to resemble a human trachea or a cat esophagus, which has rings of cartilage. (From Wyllie R, Hyams JS, editors: Pediatric Gastrointestinal and Liver Disease, ed 4. Philadelphia, Saunders, 2011,
p 398.)

82. What causes esophageal eosinophilia in EoE?
EoE is an antigen-driven immune-mediated disease. Although aeroallergens have been implicated, specifically in mouse models, the ingestion of food antigens is responsible for greater than 98% of the disease. A pathological response induces a chronic inflammatory infiltrate in the esophagus with hyperplasia of the epithelia and muscular layers and fibrosis of the lamina propria.

Virchow JC: Eosinophilic esophagitis: asthma of the esophagus? Dig Dis 32:54–60, 2014.

83. What are the symptoms of EoE?
• Infants/toddlers: FTT, feeding issues, irritability, vomiting, regurgitation
• Children: epigastric abdominal pain, vomiting, regurgitation, heartburn, and other gastroesophageal reflux disease (GERD) symptoms, and dysphagia
• Adolescents: GERD symptoms, heartburn, dysphagia, and food impaction

Liacouras CA, Spergel J, Gober LM: Eosinophilic esophagitis: clinical presentation in children, Gastroenterol Clin North Am 43:219–229, 2014.

84. What are the therapies for EoE?
• Oral steroids are used when severe symptoms are present and can promote immediate histologic and symptomatic recovery; however, they are not used long-term for maintenance therapy.
• Topical swallowed steroids are the most common pharmacologic medications used to treat EoE. These medications are used for both the initial treatment and for maintenance therapy.
• Dietary restriction involves the removal of the offending food antigen(s).
• Elemental formula is greater than 98% effective at inducing and maintaining disease remission; however, poor palatability and cost provide some limitations.
• A six-food elimination diet (removing dairy, wheat, eggs, soy, nuts, fish/shellfish) has been shown to improve symptoms and esophageal histology in 65% to 75% of patients.
• An allergy-directed diet using skin prick tests and atopy patch tests to determine offending foods has been shown to improve symptoms and esophageal histology in 60% to 70% of patients.

Dellon ES, Liacouras CA: Advances in clinical management of eosinophilic esophagitis. Gastroenterology 147 (6):1238-1254, 2014.
Liacouras CA, Furuta GT, et al: Eosinophilic esophagitis: updated consensus recommendations for children and adults, J Allergy Clin Immunol 128:3–30, 2011.

FOOD ALLERGIES
85. What are the most common food allergies in children?
Cow milk, eggs, and peanuts account for 75% of abnormal food challenges. Soy, wheat, fish, and shellfish are also common allergens.

Lack G: Food allergy, N Engl J Med 359:1252–1260, 2008.
Food Allergy and Anaphylaxis Network: www.foodallergy.org. Accessed on Mar. 20, 2015.

86. How are adverse food reactions characterized?
• Food allergy: Ingestion of food results in hypersensitivity reactions mediated most commonly by IgE.
• Food intolerance: Ingestion of food results in symptoms not immunologically mediated, and causes may include toxic contaminants (e.g., histamine in scombroid fish poisoning), pharmacologic properties of food (e.g., tyramine in aged cheeses), digestive and absorptive limitations of host (e.g., lactase deficiency), or idiosyncratic reactions.

Guandalini S, Newland C: Differentiating food allergies from food intolerances, Curr Gastroenterol Rep 13:426–434, 2011.

87. What can be the acute manifestations of milk protein allergy in childhood?
• Angioedema
• Urticaria
• Acute vomiting and diarrhea
• Anaphylactic shock
• Gastrointestinal bleeding

88. What is the most common chronic manifestation of milk protein allergy?
Diarrhea of variable severity. Histologic abnormalities of the small intestinal mucosa have been documented, with the most severe form seen as a flat villous lesion. Protein-losing enteropathy may result from disruption of the surface epithelium. The stools of children with primary milk protein intolerance often contain blood.

Warren CM, Jhaveri S, Warrier MR, et al: The epidemiology of milk allergy in US children, Ann Allergy Asthma Immunol 110:370–374, 2013.

89. What likely condition does a birch-allergic child have who develops tongue swelling when eating an apple?
Oral allergy syndrome. In this IgE-mediated condition, allergic children develop pruritus; tingling; and swelling of the lips, palate, and tongue when ingesting certain fresh fruits and vegetables because of cross-reactivity to proteins similar to those in pollen. In this case, birch shares allergens with raw carrots, celery, and apples. Symptoms generally are limited to the mouth but occasionally can progress to anaphylaxis. Most allergens are heat labile, so this patient should be advised to stick to baked apple pie for dessert.

Mansoor DK, Sharma HP: Clinical presentations of food allergy, Pediatr Clin North Am 58:315–326, 2011.

90. Can dietary manipulation in the first few months of life reduce the risk for atopic dermatitis and food allergies?
This remains a heavily debated topic in pediatric medicine. Some observational studies have suggested that introducing complementary foods before 4 months of age might have a beneficial effect on the induction of immune tolerance. The debate will likely continue pending a large randomized, controlled intervention trial. In the meantime, infants who are at high risk for developing allergy (at least one parent or sibling with allergic disease) may benefit from certain approaches as recommended by the American Academy of Pediatrics (AAP) Committee on Nutrition.
• Exclusive breastfeeding for at least 4 months decreases the incidence of atopic dermatitis and cow milk allergy during the first 2 years of life.

• Use of hydrolyzed formulas may delay or prevent atopic dermatitis.
• Solid foods should not be introduced before 4 to 6 months of age because there is no definitive evidence to support dietary intervention before this age.

Heinrich J, Koletzko B, Koletzko S: Timing and diversity of complementary food introduction for prevention of allergic diseases. How early and how much? Expert REV Clin Immunol 10:701–704, 2014.
Greer FR, Sicherer SH, Burks AW: Effects of early nutritional interventions on the development of atopic disease in infants and children: the role of maternal dietary restriction, breastfeeding, timing of introduction of complementary foods and hydrolyzed formulas, Pediatrics 121:183–191, 2008.

91. What are the symptoms of allergic proctocolitis or milk protein intolerance in an infant?
Infants, usually between birth and 4 months of age, develop frequent, mucus-streaked, bloody stools. Abdominal pain, irritability and vomiting may also be present.
92. Does the diagnosis of allergic proctocolitis in infants usually require endoscopy? No. The diagnosis is usually made based on clinical history and physical examination without the need for an endoscopy. Infants are usually being fed a milk-based formula. Proctocolitis is treated by removing the offending food antigen (milk or dairy). Switching to a soy-based formula is usually unsuccessful because of protein cross-reactivity, so it is recommended to switch to a partially hydrolyzed protein.
If symptoms persist, an amino acid–based formula may be necessary. Breastfeeding mothers must abstain from milk and soy. Sometimes additional foods need to be excluded. It is important to council patients that it may take 3 to 6 weeks to see complete improvement in both clinical symptoms and GI bleeding.

GASTROINTESTINAL BLEEDING
93. What features on physical examination can help identify an unknown cause of GI bleeding?
See Table 7-5.

Kamath BK, Mamula P: Gastrointestinal bleeding. In Liacouras CA, Piccoli DA, editors: Pediatric Gastroenterology: The Requisites in Pediatrics. Philadelphia, 2008, Mosby, pp 87–97.

Table 7-5. Features to Identify the Cause of Gastrointestinal Bleeding
Skin Signs of chronic liver disease (e.g., spider angiomas, venous distention, caput medusae, jaundice)
Signs of coagulopathy (e.g., petechiae, purpura)
Signs of vascular dysplasias (e.g., telangiectasia, hemangiomas)
Signs of vasculitis (e.g., palpable purpura on legs and buttocks suggests Henoch-Sch€onlein purpura)
Dermatologic manifestations of IBD (e.g., erythema nodosum, pyoderma gangrenosum)
Head and neck Signs of epistaxis (especially before placing a nasogastric tube, which can induce bleeding)
Hyperpigmented spots on the lips and gums (suggests Peutz-Jeghers syndrome, which is associated with multiple intestinal polyps)
Webbed neck (suggests Turner syndrome, which is associated with gastrointestinal vascular malformations and IBD)
Lesions on buccal mucosa (suggests trauma)
Lungs Hemoptysis (tuberculosis, pulmonary hemosiderosis)
Cardiac Murmur of aortic stenosis (in adults, associated with vascular malformations of the ascending colon, although this association is not certain in children)
Abdomen Splenomegaly or hepatomegaly (suggests portal hypertension and possible esophageal varices)
Continued on following page

Table 7-5. Features to Identify the Cause of Gastrointestinal Bleeding (Continued )
Ascites (suggests chronic liver disease and possible varices) Palpable or tender loops of intestine (suggests IBD)
Joint Arthritis (Henoch-Sch€onlein purpura, IBD)
Perianal Perianal ulcerations and skin tags (suggest IBD)
Perianal abscess (suggests IBD, chronic granulomatous disease immunodeficiency) Fissure (suggests constipation)
Hemorrhoids (suggests constipation, portal hypertension) Rectal mass on digital examination (suggests polyp)
Growth Failure to thrive (IBD, Hirschsprung disease)
IBD ¼ Inflammatory bowel disease.

94. In patients with acute GI bleeding, how may vital signs indicate the extent of volume depletion?
It is important to remember that when acute bleeding occurs in children, it may take 12 to 72 hours for full equilibration of a patient’s hemoglobin to occur. Vital signs are much more useful for patient management in the acute setting (Table 7-6).

Mezoff AG, Preud’homme DL: How serious is that GI bleed? Contemp Pediatr 11:60–92, 1994.

Table 7-6. Vital Signs and Blood Volume Loss
VITAL SIGNS BLOOD VOLUME LOSS
Tachycardia without orthostasis 5-10% loss
Orthostatic changes
Pulse increases by 20 beats/min
Blood pressure decreases by 10 mm Hg >10% loss
Hypotension and resting tachycardia 30% loss
Nonpalpable pulses >40% loss

95. What is the simplest way of differentiating upper GI from lower GI bleeding? Nasogastric lavage. After the insertion of a soft nasogastric tube (12 Fr in small children, 14 to 16 Fr in older children), 3 to 5 mL/kg of room-temperature normal saline is instilled. If bright red blood or coffee- ground–like material is aspirated, the test is positive. A pink-tinged effluent is not a positive test because it can simply denote the dissolution of a clot and not active intestinal bleeding. By definition, upper
GI bleeding occurs proximal to the ligament of Treitz. If the lavage is negative, it is unlikely that the bleeding is above this ligament, and this rules out gastric, esophageal, or nasal sources. However, bleeding from duodenal ulcers and duodenal duplications may sometimes be missed by these aspirates.
96. How does the type of bloody stool help pinpoint the location of a GI bleed?
• Hematochezia (bright red blood): Normal stool spotting on toilet tissue likely suggests distal bleeding (e.g., anal fissure, juvenile colonic polyp). Mucous or diarrheal stools (especially if painful) indicate left-sided or diffuse colitis.
• Melena (black, tarry stools) indicates blood denatured by acid and usually implies a lesion, likely before the ligament of Treitz. However, melena can be seen in patients with Meckel diverticulum as a result of denaturation by anomalous gastric mucosa.
• Currant jelly (dark maroon) stools usually come from the distal ileum or colon and often are associated with ischemia (e.g., intussusception).
Because blood is a cathartic, intestinal transit time can be greatly accelerated and makes defining the site of bleeding by the magnitude and color of the blood difficult. This difficulty underscores
the importance of the initial nasogastric tube insertion.

97. What can cause false-negative and false-positive results when stool testing for blood?
Hemoglobin and its various derivatives (e.g., oxyhemoglobin, reduced hemoglobin, methemoglobin, carboxyhemoglobin) can serve as catalysts for the oxidation of guaiac (Hemoccult) or benzidine (Hematest) when a hydrogen peroxide developer is added, thereby producing a color change.
Of note, iron does not cause false-positive results.
False negatives: Ingestion of large doses of ascorbic acid; delayed transit time or bacterial overgrowth, allowing bacteria to degrade the hemoglobin to porphyrin
False positives: Recent ingestion of red meat or peroxidase-containing fruits and vegetables (e.g., broccoli, radishes, cauliflower, cantaloupes, turnips)
98. How do the causes of lower GI bleeding vary by age group?
Newborn and infant:
• Mucosal: Peptic ulcer disease, necrotizing enterocolitis, infectious colitis, eosinophilic or allergic colitis, Hirschsprung enterocolitis, anal fissure
• Structural: Intestinal duplication, Meckel diverticulum, intussusception
Child:
• Mucosal: Anal fissure, juvenile polyp, infectious colitis, inflammatory bowel disease, solitary rectal ulcer, lymphonodular hyperplasia
• Structural: Intestinal duplication, Meckel diverticulum, intussusception, volvulus, Dieulafoy malformation (large tortuous arteriole in the stomach that erodes and bleeds)
• Other: Hemolytic-uremic syndrome, Henoch-Sch€onlein purpura, Munchausen syndrome by proxy, arteriovenous malformation, vascular malformation

Kamath BK, Mamula P: Gastrointestinal bleeding. In Liacouras CA, Piccoli DA, editors: Pediatric Gastroenterology: The Requisites in Pediatrics. Philadelphia, 2008, Mosby, pp 87–97.

99. A previously asymptomatic 18-month-old child has large amounts of painless rectal bleeding (red but mixed with darker clots). What is the likely diagnosis?
Although juvenile polyps can also cause painless rectal bleeding, the likely diagnosis is a Meckel diverticulum. This outpouching occurs from the failure of the intestinal end of the omphalomesenteric duct to obliterate. Up to 2% of the population may have a Meckel diverticulum, and about half contain gastric mucosa; most are usually silent throughout life. Meckel diverticulum is twice as common in males and usually appears during the first 2 years of life as massive
painless bleeding that is red or maroon in color. Tarry stools are observed in about 10% of cases. A history of previous minor episodes may be obtained. The presentation can range from shock to intussusception with obstruction, volvulus, or torsion. Meckel diverticulitis, which occurs in 10% to 20% of cases, may be indistinguishable from appendicitis.

100. Worldwide, what is the most common cause of GI blood loss in children? Hookworm infection. Caused by the parasites Necator americanus and Ancylostoma duodenale, this infection is often asymptomatic. Progressive microscopic blood loss often leads to anemia as a result of iron deficiency.

Crompton DW: The public health importance of hookworm disease, Parasitology 121:S39–S50, 2000.

101. How do the causes of upper GI bleeding vary by age group?
• Newborns: Swallowed maternal blood, vitamin K deficiency, stress gastritis or ulcer, vascular anomaly, coagulopathy, milk-protein sensitivity
• Infants: Stress gastritis or ulcer, acid-peptic disease, Mallory-Weiss tear, vascular anomaly, GI duplications, gastric or esophageal varices, duodenal or gastric webs, bowel obstruction
• Children: Mallory-Weiss tear, acid-peptic disease, varices, caustic ingestion, vasculitis, hemobilia, tumor

Gilgar MA: Upper gastrointestinal bleeding. In Walker WA, Goulet O, Kleinman RE, et al, editors: Pediatric Gastrointestinal Disease, ed 4. Hamilton, Ontario, 2004, BC Decker, pp 258–265.

102. What is the most likely cause of hematemesis in a healthy term infant? Swallowed maternal blood. The Apt test can be used to differentiate maternal from infant blood. Fetal hemoglobin resists denaturation with alkali better than adult hemoglobin does. Therefore, exposure of adult blood to sodium hydroxide will result in a brown color, whereas the newborn infant’s blood will remain pink.
103. What are the two most likely causes of visible blood in the stool of an otherwise healthy infant?
Anal/rectal fissure and milk/soy protein allergy. A physical exam and rectal exam are of particular importance in making the diagnosis.

KEY POINTS: GASTROINTESTINAL BLEEDING
1. Hemoglobin measurement is a much less reliable indicator of volume depletion than vital signs during the assessment of acute gastrointestinal bleeding.
2. Nasogastric lavage is a simple method for differentiating upper gastrointestinal bleeding from lower gastrointestinal bleeding and should always be performed in all patients suspected of having a significant gastrointestinal bleed.
3. The two most common causes of painless rectal bleeding in children are juvenile polyps and Meckel diverticulum.

104. What are the six most common causes of massive GI bleeding in children?
1. Esophageal varices
2. Meckel diverticulum
3. Hemorrhagic gastritis
4. Crohn disease with ileal ulcer
5. Peptic ulcer (mainly duodenal)
6. Arteriovenous malformation

Treem WR: Gastrointestinal bleeding in children, Gastrointest Endosc Clin North Am 5:75–97, 1994.

GASTROINTESTINAL DYSMOTILITY
105. How rapidly do infants outgrow GER?
Forty percent of healthy infants have spitting or regurgitation more than once a day; mild
reflux does not represent disease. As a rule, in those infants who have more significant primary GER (about 12% of total), 25% to 50% resolve by 6 months of age, 75% to 85% by 12 months of age, and 95% to 98% by 18 months of age. GER in older children may be more widespread
than appreciated. Surveys of parents of children and adolescents (3 to 17 years) revealed that symptoms of heartburn regurgitation were relatively common (2% to 8% of patients).

Campanozzi A, Boccia G, Pensabene L, et al: Prevalence and natural history of gastroesophageal reflux: pediatric prospective study, J Pediatr 123:779–783, 2009.

106. When does GER become GERD (gastroesophageal reflux disease)?
GERD occurs when physiologic GER (a variation of normal; “happy spitters”) becomes pathologic with the onset of symptoms and complications. These could include feeding refusal, poor
weight gain, painful emesis, chronic respiratory problems, and others. The delineation can be imprecise, and other medical conditions can present with symptoms similar to GERD or with secondary GERD.

Grossman AB, Liacouras CA: Gastrointestinal bleeding. In Liacouras CA, Piccoli DA, editors: Pediatric Gastroenterology: The Requisites in Pediatrics. Philadelphia, 2008, Mosby, pp 74–86.

107. What are the diagnostic methods for GER?
The diagnosis can be made either clinically or by diagnostic testing. Clinically, reflux should be suspected in any child who demonstrates frequent, effortless vomiting or regurgitation without evidence of GI obstruction. Clinical response to medical therapy can be diagnostic.
The upper GI barium study does not reliably indicate reflux but can assess for anatomic abnormalities, such as malrotation, which might contribute. Nuclear scintigraphy, a noninvasive test that uses radiolabeled milk (“milk scan”) or a meal, can detect postprandial reflux and delay in gastric emptying but cannot distinguish between physiologic and pathologic reflux. The presence of histologic esophagitis on an endoscopic examination is suggestive but not diagnostic of reflux; the absence of esophagitis does not rule out reflux. The 24-hour pH probe, traditionally thought
to be the most reliable test for the diagnosis of GER, only detects acid reflux and cannot detect nonacid reflux. Multichannel intraluminal impedance is a newer technology that can be performed with a pH probe to assess all types of reflux: acid, weakly acid, and alkaline. It is often used in combination with pH probe testing to separate acid from nonacid reflux.

Shin MS: Esophageal pH and combined impedance-pH monitoring in children, Pediatr Gastroenterol Hepatol Nutr
17:13–22, 2014.
van der Pol RJ, Smits MJ, Venmans L, et al: Diagnostic accuracy of tests in pediatric gastroesophageal reflux disease,
J Pediatr 162:983–987, 2013.

108. How effective are nonpharmacologic agents as treatments for suspected GER? They can be quite effective clinically. In a study of infants with suspected GER, the following changes resulted in improvements after 2 weeks in reflux scores in three quarters and normalization in one quarter of patients:
• Switching formula-fed infants to semi-elemental formula thickened with rice cereal
• If breastfeeding, the mother should eliminate cow milk and soy products from her diet
• Avoiding seated and supine positioning as much as possible for the infant, especially after feeding
• Eliminating tobacco smoke because of its association with increased GERD

Orenstein SR, McGowan JD: Efficacy of conservative therapy as taught in the primary care setting for symptoms suggesting infant gastroesophageal reflux, J Pediatr 152: 310–314, 2008.

109. How effective are H2 blockers and proton pump inhibitors (PPIs) in the treatment of GER?
These medications, though widely prescribed, have very limited data on efficacy in the treatment of GER. Part of the lack of demonstrated clinical success may be that many symptoms blamed on GER (such as cough, gagging, desaturations, back arching, fussiness, and pain) and treated with H2 antagonists or PPIs are not associated with reflux events when studied with pH-multichannel intraluminal probes.

van der Pol R, Langendam M, Benninga M, et al: Efficacy and safety of histamine-2 receptor antagonists, JAMA Pediatr 168(10):947–54, 2014.
Chen I-L, Gao W-Y, et al: Proton pump inhibitor use in infants: FDA reviewer experience, J Pediatr Gastroenterol Nutr 54:8–14, 2012.

110. An infant with known GER who periodically arches his or her back may have what syndrome?
Sandifer syndrome is paroxysmal dystonic posturing with opisthotonus and unusual twisting of the head and neck (resembling torticollis) in association with GER. Typically, an esophageal hiatal hernia is also present.
111. What is a Nissen fundoplication?
Nissen fundoplication is the most commonly performed antireflux surgical procedure. It involves wrapping a portion of the gastric fundus 360 degrees around the distal esophagus in an effort to tighten the gastroesophageal junction.

112. Which patients are candidates for fundoplication?
Most infants with developmental GER do not require fundoplication. It is indicated in patients with recurrent aspiration, refractory or Barrett esophagitis, reflux-associated apnea, and reflux- associated FTT that is refractory to medical therapy. Patients with severe reflux and psychomotor retardation should be evaluated for fundoplication if a feeding gastrostomy is contemplated.

KEY POINTS: GASTROESOPHAGEAL REFLUX
1. More than 40% of healthy infants regurgitate effortlessly more than once per day. This does not represent significant gastroesophageal reflux.
2. Gastroesophageal reflux disease is usually a clinical diagnosis. Testing, such as upper gastrointestinal testing, nuclear scintigraphy, pH and impedance monitoring, and upper endoscopy, can be helpful in certain cases, but it usually is not necessary.
3. By the age of 12 months, the symptoms of 95% of infants with significant reflux have resolved.

113. A teenage girl has symptoms of swallowing difficulties improved by positional head and neck changes, nocturnal regurgitation, and halitosis. What is the leading diagnosis? Achalasia, which is a motor disorder of the esophagus characterized by loss of esophageal peristalsis, increased lower esophageal sphincter (LES) pressure, and absent or incomplete relaxation of LES with swallowing. Most cases are sporadic, and patients can present at any age from birth until the ninth decade of life. Suspected causes include autoimmune, infectious, and environmental triggers.
114. What are the key tests to diagnose achalasia?
A barium swallow/video-esophagram will show a variable degree of esophageal dilatation with tapering at the gastroesophageal junction. Figure 7-9 shows the characteristic “birds beak” esophagus. Later in the disease process, the proximal esophagus can become widely dilated and tortuous and plain chest x-ray may show a widened mediastinum. Esophageal manometry (esophageal motility study) measures the pressure generated by the esophageal muscle. It can detect achalasia earlier in its course when a video-esophagram may be normal.

Figure 7-9. Barium swallow in a child with achalasia showing esophageal dilation and rapid tapering in a beaklike appearance. (From Wyllie R, Hyams JS, Kay M, editors: Pediatric Gastrointestinal and Liver Disease, ed 3. Philadelphia, 2006, Saunders, p 330.)

115. What are treatment options for achalasia?
Treatment options include pneumatic dilations (via therapeutic endoscopy), corrective laparoscopic surgery, botulinum toxin injection at the lower esophageal sphincter, and pharmacologic therapies. Pneumatic dilation is relatively well tolerated, but it often needs to be repeated if symptoms recur. Regardless of treatment modality used, patients continue to be at increased risk for aspiration

secondary to pooling of food and saliva in the esophagus after meals. Many have complications of reflux esophagitis, which require ongoing surveillance.

Lee CW, Kays DW, Chen MK, et al: Outcomes of treatment of childhood achalasia, J Pediatr Surg 45:1173–1177, 2010.

116. What are the common symptoms of gastroparesis?
Gastroparesis is a disorder of gastric motility characterized by impairment of gastric contraction and emptying. Common symptoms include bloating, early satiety, nausea, vomiting (especially of undigested food eaten many hours before), and abdominal discomfort in the absence of
mechanical obstruction.
117. In what clinical settings should gastroparesis be suspected?
• Preterm infants with immature GI tract
• Infants with cow milk protein allergy
• Postinfectious, including viral (rotavirus, Epstein-Barr virus [EBV], cytomegalovirus [CMV]) and
Mycoplasma infection
• Postsurgical, including vagal nerve injury in upper abdominal surgery such as fundoplication or bariatric surgery
• Cystic fibrosis
• Type 1 diabetes mellitus
• Chronic intestinal pseudo-obstruction
• Muscular dystrophy
• Systemic autoimmune disorders, such as scleroderma
118. How is postinfectious gastroparesis diagnosed?
Patients will commonly present with persistent vomiting for days, weeks, or even months after a viral illness. Often the acute illness has passed, and the offending pathogen cannot be isolated. Diagnosis is mainly clinical, but can be confirmed with a delayed gastric emptying scan.

Saliakellis E, Fotoulaki M: Gastroparesis in children, Ann Gastroenterol 26: 204–211, 2013.

119. How is gastroparesis treated?
Dietary and behavioral modifications: Avoidance of carbonated beverages which can distend the stomach, drinking fluids throughout a meal, and walking 1 to 2 hours after a meal can promote better stomach emptying. In severe cases, a majority of calories can be provided in liquid form.
Pharmacotherapy: Prokinetic agents including (1) dopamine receptor antagonists (e.g., metoclopramide), which increase duration and frequency of antral and duodenal contractions, increase LES pressure, and relax the pyloric sphincter, and (2) erythromycin, a macrolide antibiotic with agonist activity of motilin receptors in smooth muscle cells of the GI tract (stomach and small bowel).
120. A 12-year-old who presents with weight loss and a history of effortlessly and involuntarily regurgitating many meals has what likely diagnosis? Rumination syndrome. This is a functional gastrointestinal motility disorder characterized by repetitive effortless regurgitation of recently swallowed food from the stomach into the mouth within 30 minutes of ingesting the meal. When the stomach contents reach the mouth, it is either reswallowed or expelled. In infants and young children, rumination is commonly seen in patients with neurologic impairment or developmental delay. Adolescents are typically healthy. Children who have rumination typically do not retch and do not complain of dyspeptic/heartburn symptoms. Rumination syndrome can be difficult to diagnose. Differential diagnoses include bulimia nervosa and gastroparesis. The most effective treatments involve biofeedback and relaxation techniques.

Kessing BF, Smout AJ, Bredenoord AJ: Current diagnosis and management of the rumination syndrome, J Clin Gastroenterol 48:478-483, 2014.

HEPATIC, BILIARY AND PANCREATIC DISEASE
121. What laboratory tests are commonly used to evaluate liver disease?
See Table 7-7.

Table 7-7. Laboratory Tests Commonly Used to Evaluate Liver Disease
TEST CLINICAL SIGNIFICANCE
Alanine aminotransferase (ALT, SGPT) Increased with damaged hepatocytes
Aspartate aminotransferase (AST, SGOT) Less sensitive than ALT for hepatic injury
Alkaline phosphatase (AP) Increased in cholestatic disease; also comes from bone Higher in children because of bone growth (can identify source through
isoenzyme)
γ-Glutamyltransferase (GGT) More sensitive marker for cholestasis than AP
Bilirubin Differential diagnosis different for conjugated versus unconjugated
Albumin Low albumin can indicate chronic impairment in hepatic synthetic function
Prealbumin Shorter half-life; may reflect more acute synthetic capabilities
Prothrombin time (PT) Reflects synthetic function as a result of short half-life of factors
Ammonia Impaired removal in patients with chronic liver disease; can lead to encephalopathy

122. What conditions are associated with elevations of aminotransferases?
• Steatosis (fatty liver due to metabolic syndrome)
• Hepatocellular inflammation (hepatitis)
• Drug- or toxin-associated hepatic injury
• Hypoperfusion or hypoxia
• Passive congestion (right-sided congestive heart failure, Budd-Chiari syndrome, constrictive pericarditis)
• Nonhepatic disorders (muscular dystrophy, celiac disease, macroenzyme of aspartate aminotransferase)

Teitelbaum JE: Normal hepatobiliary function. In Rudolph CD, Rudolph AM, editors: Rudolph’s Pediatrics, ed 21. New York, 2003, McGraw-Hill, pp 1479.

123. What is the most frequent cause of chronically elevated aminotransferases among children and adolescents in the United States?
Nonalcoholic fatty liver disease (NAFLD). The condition is most commonly associated with the metabolic syndrome in obese patients. Hepatic steatosis (abnormal lipid deposition in hepatocytes) occurs in the absence of excess alcohol intake. A main concern of the condition is that this simple benign fatty liver may progress to nonalcoholic steatohepatitis (or NASH) which involves inflammation of the liver and hepatocellular damage. This ultimately can lead to cirrhosis with possible liver failure and portal hypertension. It is unclear how and why certain children make that significant pathologic jump to marked liver disease.

Berardis S, Sokal E: Pediatric non-alcoholic fatty liver disease: an increasing public health issue. Eur J Pediatr
173:131–139, 2014.

124. What is the main reason for the apparent increase in pediatric NAFLD?
The growing obesity epidemic is likely responsible. Some studies indicate that about half of obese children may have fatty liver. Children of Asian descent and of Hispanic (mostly Mexican) descent are also at increased risk compared with white and black American children.

Nieregarten MB, Freed GL: Pediatric nonalcoholic fatty liver disease, Contemp Pediatr 30:14, 2013.

125. What is the best way to screen for NAFLD?
Established screening guidelines are currently lacking. The most widely used test is serum alanine aminotransferase (ALT), which is the most common abnormal laboratory finding noted in the disease, but the sensitivity is low. In addition, the height of the measurement does not correlate with disease severity. Conversely, normal levels do not exclude possible fibrosis.
Imaging studies, particularly ultrasound, have some diagnostic merit with potential detection of increased fatty echogenicity and hepatic enlargement. However, ultrasound is diagnostically most effective when hepatic steatosis is more advanced with a liver fat content of >30%. Computed tomography (CT) is a better study, but it has obvious radiation implications. The gold standard for diagnosis, liver biopsy, is too invasive as a screening test.

Berardis S, Sokal E: Pediatric non-alcoholic fatty liver disease: an increasing public health issue. Eur J Pediatr
173:131–139, 2014.

126. Why is it important to determine whether an elevated bilirubin is conjugated or unconjugated?
Bilirubin released from erythrocytes (unconjugated) is taken up by the liver and enzymatically converted (conjugated) to a more water-soluble form. On the basis of laboratory methodology, measurements of unconjugated bilirubin are referred to as indirect reacting and those of conjugated bilirubin as direct reacting. Elevated conjugated bilirubin is associated with obstruction of the biliary tract, intrahepatic cholestasis, or poorly functioning hepatocytes. Conjugated hyperbilirubinemia always requires further evaluation.

Harb R, Thomas DW: Conjugated hyperbilirubinemia: screening and treatment in older infants and children, Pediatr REV 28:83–90, 2007.

127. When are levels of conjugated bilirubin considered abnormal?
Levels >20% of total bilirubin are considered abnormal. In significant indirect (unconjugated) hyperbilirubinemia, direct (conjugated) levels usually do not exceed 15%. The levels between 15% and 20% are thus somewhat indeterminate. Generally, direct bilirubin does not exceed more than 2 mg/dL.
128. What are the common causes of neonatal hepatitis and neonatal cholestasis?
See Table 7-8.

Table 7-8. Neonatal Conjugated Hyperbilirubinemia and Neonatal Hepatitis
Neonatal Hepatitis Metabolic
Idiopathic α1-Antitrypsin deficiency
Viral Tyrosinemia
Cytomegalovirus Galactosemia
Herpesviruses Cystic fibrosis
Hepatitis viruses Bile acid synthetic disorders
Human immunodeficiency virus Storage disorders
Enterovirus Niemann-Pick disease
Rubella Gaucher disease
Adenovirus Lipidoses
Bacterial Peroxisomal disorders
Bile Duct Obstruction Endocrine
Biliary atresia Hypothyroidism
Choledochal cyst Panhypopituitarism
Neonatal sclerosing cholangitis Other Inherited Causes
Congenital hepatic fibrosis Alagille syndrome
Cholelithiasis Familial intrahepatic cholestasis
Tumor or mass Neonatal iron storage disease
Toxic
Parenteral nutrition
Drugs
Cardiovascular Disorders
Adapted from Suchy FJ: Approach to the infant with cholestasis. In Suchy FJ, Sokol RJ, Balistreri WF, editors: Liver Disease in Children, ed 3. New York, 2007, Cambridge UNIVERSITY Press, pp 179–189.

129. What is the likelihood of chronic hepatic disease developing after acute infections with hepatitis viruses A to G?
• Hepatitis A: 95% recover within 1 to 2 weeks of illness; chronic disease is unusual
• Hepatitis B: >90% of perinatally infected infants develop chronic hepatitis B infection; 25% to 50% of children who acquire the virus between 1 and 5 years of age develop chronic infection; in older children and adults, only 6% to 10% develop chronic infection
• Hepatitis C: 50% to 60% develop persistent infection
• Hepatitis D: Occurs only in patients with acute or chronic hepatitis B infection; 80% develop viral persistence
• Hepatitis E: Does not cause chronic hepatitis
• Hepatitis G: Unknown

American Academy of Pediatrics: Hepatitis A-G. In Pickering LK, editor: 2012 Red Book, Report of the Committee on Infectious Diseases, ed 29. Elk Grove Village, IL, 2012, American Academy of Pediatrics, pp 361–395.

130. Other than viral hepatitis, what are other common causes of acute and chronic hepatitis in children?
• Metabolic and genetic disorders: Wilson disease, α1-antitrypsin deficiency, cystic fibrosis, steatohepatitis
• Toxic hepatitis: Drugs, hepatotoxins, radiation
• Autoimmune: Autoimmune hepatitis, primary sclerosing cholangitis: anti–smooth muscle antibody positive, anti–liver-kidney-microsomal antibody positive
• Anatomic: Cholelithiasis, choledochal cyst
• Other infectious: CMV, EBV
• Toxic: Ethanol, acetaminophen
• Other inherited: Alagille syndrome, cystic fibrosis, familial intrahepatic cholestasis
131. How is α1-antitrypsin deficiency most likely to present in infants and children? α1-Antitrypsin deficiency is an autosomal recessive disorder that causes lung and liver disease. In the liver, injury results from intracellular accumulation of the mutant α1-antitrypsin protein.
In the lungs, the absence of functional α1-antitrypsin leads to unchecked leukocyte elastase function, resulting in destruction of the alveolar walls and eventual emphysema. The pulmonary effects take years to evolve, so lung disease rarely is present in children. More common presenting symptoms are neonatal cholestasis, hepatomegaly, and chronic hepatitis.
Although most patients do not have severe disease, this can progress to cirrhosis with liver failure.

Stockley RA: Alpha 1-antitrypsin deficiency, Clin Chest Med 35:39–50, 2014.

132. Why is measuring the level serum level of α1-antitrypsin not enough to diagnose α1-antitrypsin deficiency?
α1-Antitrypsin is an acute phase reactant and might not be decreased in all cases of α1-antitrypsin deficiency. Pi typing (short for protease inhibitor typing) by electrophoresis is necessary to make the diagnosis. MM is the normal phenotype and has the highest activity; ZZ has the lowest activity and the most common association with liver disease. PiMM is the most common Pi type, with a distribution of about 87%; PiMS represents 8% and PiMZ 2%. The incidence of PiZZ ranges between 1 in 2000 and 1 in 5000.

Silverman EK, Sandhaus RA: Alpha1-antitrypsin deficiency, N Engl J Med 360:2749–2757, 2009.

133. What is the metabolic defect in patients with Wilson disease?
Wilson disease is an autosomal recessive defect of copper metabolism that results in markedly increased levels of copper in many tissues, most notably the liver, basal ganglia, and cornea (Kayser-Fleischer rings). The primary defect is a mutation in the transmembrane protein ATP7B, which is key to excreting excess copper into the biliary canalicular system. The combination of

markedly increased copper levels in a liver biopsy specimen, low serum ceruloplasmin, and increased urinary copper excretion strongly suggests Wilson disease.

Ala A, Walker AP, Ashkan K, et al: Wilson’s disease, Lancet 369:397–408, 2007.

134. What are the treatments of choice for Wilson disease?
Copper-chelating agents. D-Penicillamine has traditionally been the drug of choice, but another chelator, trientine, has been used successfully in patients who have discontinued penicillamine because of hypersensitivity reactions. Some advocate for trientine as an alternative agent to penicillamine because trientine has a better safety profile. Zinc sulfate, which inhibits intestinal copper absorption, has also been used. Patients require a low copper diet for life.

135. A 3-year-old child who experiences mild fluctuating jaundice in times of illness
“just like his Uncle Kevin” is likely to have what condition?
Gilbert syndrome, which is due primarily to a decrease in hepatic glucuronyl transferase activity. Normally, bilirubin is disconjugated to glucuronic acid. In patients with Gilbert syndrome, the defective total conjugation results in the increased production of monoglucuronides in bile and mild elevation in serum unconjugated (indirect) bilirubin. The syndrome is inherited in an autosomal dominant fashion with incomplete penetrance (boys outnumber girls by 4 to 1).
Frequency of this gene in the population is estimated at 2% to 6%. Elevations of bilirubin are noted during times of medical and physical stress, particularly fasting.

136. What are the clinical findings of portal hypertension?
Obstruction of portal flow is manifested by two physical signs: splenomegaly and increased collateral venous circulations. Collaterals are evident on physical examination in the anus and abdominal wall and by special studies in the esophagus. Hemorrhoids may suggest collaterals, but, in older patients, these are present in high frequency without liver disease, and thus their presence has no predictive value. Dilation of the paraumbilical veins produces a rosette around the umbilicus (the caput medusae), and the dilated superficial veins of the abdominal wall
are visible. A venous hum may be present in the subxiphoid region from varices in the falciform ligament.

137. How does autoimmune hepatitis (AIH) typically present?
There are three typical patterns of presentation: (1) acute hepatitis, with nonspecific symptoms of malaise, nausea and vomiting, anorexia, jaundice, dark urine, and pale stools; (2) insidious,
with progressive fatigue, relapsing jaundice, headache, and weight loss; and (3) despite no history of jaundice, patients present with complications of portal hypertension (splenomegaly, GI bleeding from varices, and weight loss). Type I AIH is more common and characterized by antineutrophil antibodies and anti–smooth muscle antibodies. Type 2 AIH is characterized by anti–liver-kidney- microsomal antibodies.
138. A patient with liver failure develops confusion. Why worry?
Hepatic encephalopathy can appear as either a rapid progression to coma or as mild fluctuations in mental status over an extended amount of time. A single underlying cause has not been established, but suspected toxins include ammonia, other neurotoxins, and relatively increased γ-aminobutyric acid activity. Management requires the limitation of protein intake, the use of lactulose to promote mild diarrhea, antibiotics to reduce ammonia production, intracranial pressure monitoring in advanced cases, and possible peritoneal dialysis for patients in severe coma and before liver transplantation.
139. What is the most common indication for pediatric liver transplantation?
The most common indication is extrahepatic biliary atresia with chronic liver failure after a Kasai hepatoportoenterostomy. Other common indications include inborn errors of metabolism (e.g., α1- antitrypsin deficiency, hereditary tyrosinemia, Wilson disease) and idiopathic fulminant hepatic failure.
140. Calculous and acalculous cholecystitis: what are the differences?
Calculous cholecystitis: Gallstone impaction in the cystic duct results in gallbladder distention edema, biliary stasis, and bacterial overgrowth (e.g., E. coli, Klebsiella, enterococci). If untreated, this can lead to gallbladder infarction, gangrene, and perforation.

Acalculous cholecystitis: Gallbladder dysfunction results from a variety of conditions including major trauma, sepsis/hypotension, and diabetes. Bile stasis results, which can lead to an inflammatory response; ischemia; distention; and eventually, necrosis of gallbladder tissue.
141. Which patients are at risk for cholelithiasis?
See Table 7-9.

Table 7-9. Patients at Risk for Cholelithiasis
PIGMENT STONE CHOLESTEROL STONE
Race — Native American
Sex — Female
Age — Adolescence
Diet — Obesity
Total parenteral nutrition +++ —
Hemolytic disease (especially sickle-cell disease, thalassemia, hereditary spherocytosis) +++ —
Cystic fibrosis — +++
Ileal disease — +++
Defects in bile salt synthesis — +++
Hypertriglyceridemia — +++
Diabetes mellitus — +++
+++¼ increased risk

142. What are the possible causes of pancreatitis in children?
In adults, the majority of cases of pancreatitis arise from gallstones or alcohol. In children, there is a much greater diversity in etiology.
• 33%: Systemic disorders (sepsis and shock, vasculitis)
• 13% to 34%: Idiopathic
• 10% to 40%: Trauma (motor vehicle accidents, sports injuries, accidental falls, child abuse)
• 10% to 30%: Biliary disease (gallstones, sludge)
• <25%: Medications (valproic acid, L-asparaginase, prednisone, 6-mercaptopurine)
• <10%: Infections (various viral, including mumps)
• 5% to 8%: Hereditary (genetic mutations)
• 2% to 7%: Metabolic disorders (diabetic ketoacidosis [DKA], hypertriglyceridemia, hypercalcemia)
• <3%: Anatomic and structural anomalies (pancreatic divisum, duct anomalies, sphincter of Oddi dysfunction)

Bai HX, Lowe ME, Husain SZ: What have we learned about acute pancreatitis in children? J Pediatr Gastroenterol Nutr
52:262–270, 2011.

143. What is the typical presentation of acute pancreatitis in children?
In children >3 years of age, the most common symptom is abdominal pain, which occurs in 80% to 95%. Pain is typically epigastric in location. Radiation to the back is uncommon. Nausea or vomiting occurs in 40% to 80%. Abdominal distention is also common. Infants and toddlers are less likely to
complain of abdominal pain and nausea and are more likely to have fever.

Bai HX, Lowe ME, Husain SZ: What have we learned about acute pancreatitis in children? J Pediatr Gastroenterol Nutr
52:262–270, 2011.

144. Which enzyme is a more sensitive marker of pancreatic injury in children: amylase or lipase?
There is no clear winner. In a compilation of pediatric studies, the sensitivity of the amylase test in diagnosing pancreatitis has ranged from 50% to 85%, while lipase was only marginally more sensitive than amylase in most studies. Amylase values rise 2 to 12 hours after the onset of pancreatitis; lipase values rise at 4 to 8 hours. Because only one or the other may be elevated in individual patients, both lipase and amylase should be measured in suspected pancreatitis.

Srinath AI, Lowe ME: Pediatric pancreatitis, Pediatr REV 34:79–89, 2013.
Bai HX, Lowe ME, Husain SZ: What have we learned about acute pancreatitis in children? J Pediatr Gastroenterol Nutr
52:262–270, 2011.

145. What conditions may be associated with hyperamylasemia?
Pancreatic: pancreatitis, pancreatic tumors, pancreatic duct obstruction, biliary obstruction, perforated ulcer, bowel obstruction, acute appendicitis, mesenteric ischemia, endoscopic retrograde cholangiopancreatogram (ERCP)
Salivary: infections (mumps), trauma, salivary duct obstruction, lung cancer, ovarian tumors or cysts, prostate tumors, DKA
Mixed or unknown: cystic fibrosis, renal insufficiency, pregnancy, cerebral edema, burns

KEY POINTS: HEPATIC AND BILIARY DISEASE
1. Portal hypertension manifests clinically as splenomegaly and increased collateral venous circulation.
2. Conjugated hyperbilirubinemia in any child is abnormal and deserves further investigation.
3. Extrahepatic biliary atresia is the most common pediatric indication for liver transplantation.
4. The younger the patient, the more likely it is that acute hepatitis B infection will become chronic.

INFLAMMATORY BOWEL DISEASE
146. What is the epidemiology of pediatric inflammatory bowel disease (IBD)?
The incidence and prevalence of IBD in general, and Crohn disease specifically, has increased over recent decades. The most significant increases have occurred in younger children with increases of 5% in those <4 years old and nearly 8% in children 5 to 9 years old. About 20% of all IBD cases are diagnosed before age 10 years. Mean age of diagnosis of pediatric IBD is 12½ years. About 10,000 new cases are diagnosed annually. Up to 25% of children who are diagnosed with IBD have a positive family history.

Glick SR, Carvalho RS: Inflammatory bowel disease, Pediatr REV 32:14–24, 2011.

147. How do ulcerative colitis and Crohn disease vary in intestinal distribution? Ulcerative colitis is limited to the superficial mucosa of the colon. It always involves the rectum and extends proximally to a variable extent. Ulcerative colitis more commonly involves the entire colon in children than in adults, who more commonly will have limited left-sided disease. Regional enteritis, or Crohn disease, is characterized by transmural inflammation of the bowel that may affect the entire tract from the mouth to the anus. Because of the transmural nature of the inflammation, patients can develop fistulas and abscesses more commonly with Crohn disease. The typical cobblestone appearance of Crohn disease is produced by crisscrossing ulcerations (Fig. 7-10). Crohn colitis, with no involvement of the small bowel, is more common in younger children and can be difficult to distinguish from ulcerative colitis.

Abraham C, Cho JH: Inflammatory bowel disease, N Engl J Med 36:2066–2078, 2009.
Bousvaros A, Antonioli DA, Colletti RB, et al: Differentiating ulcerative colitis from Crohn disease in children and young adults, J Pediatr Gastroenterol Nutr 44:653–674, 2007.

148. What features differentiate ulcerative colitis from Crohn disease?
See Table 7-10.

Crohn’s and Colitis Foundation of America: www.ccfa.org. Accessed Nov. 24, 2014.

Figure 7-10. Crisscrossing ulcerations produce a cobblestone appearance in patients with Crohn disease. (From Katz DS, Math KR, Groskin SA: Radiology Secrets. Philadelphia, 1998, Hanley & Belfus, p 150.)

Table 7-10. Features that Differentiate Ulcerative Colitis from Crohn Disease
ULCERATIVE COLITIS CROHN DISEASE
Distribution Colon only (gastritis recognized) Continuous Entire gastrointestinal tract Skip lesions
Clinical presentation
Bleeding Very common Common
Growth failure Uncommon Common
Weight loss Less common Common
Obstruction Uncommon Common
Perianal disease Rare Common
Endoscopic findings Continuous inflammation 100% rectal involvement Erythema, edema, friability,
ulceration on abnormal mucosa Focal or segmental inflammation Rectal sparing
Aphthous or linear ulcerations on normal-appearing mucosa
Cobblestoning
Abnormal terminal ileum: >50%
Histologic findings Mucosa only No granulomas Full-thickness granulomas

149. What is the role of serologic panels in the diagnosis of IBD?
Certain antibodies can help distinguish between Crohn disease and ulcerative colitis in patients with indeterminate colitis. These panels are not useful in population screening as false-positive results can create unwarranted anxiety and unnecessary testing. One-third of individuals with positive serology do not have IBD. Higher antibody titers are associated with more aggressive disease. Average positivity is as follows:
• ASCA (anti-Saccharomyces CEREVISIAE antibody): CD (40% to 56%), UC (0% to 7%), Controls (<5%)

• ANCA (anti-neutrophil cytoplasmic antibody): CD (18% to 24%), UC (60% to 80%), Controls (<5%)
• Anti-OmpC (outer membrane protein C, E. coli): CD (25%), UC (6%), Controls (3%)

Glick SR, Carvalho RS: Inflammatory bowel disease, Pediatr REV 32:18–19, 2011.

150. What are the extraintestinal manifestations of pediatric IBD?
In addition to the typical gastrointestinal involvement, other organ systems can become involved in IBD. These manifestations may become the major source of morbidity and presenting symptom(s) for some patients.
• Growth failure
• Arthralgias/arthritis
• Bone disease including osteopenia and osteoporosis
• Oral lesions, most commonly recurrent aphthous lesions
• Skin lesions: granulomatous, reactive, and secondary to nutritional deficiencies
• Eye lesions: episcleritis and uveitis
• Liver disease: hepatitis, fatty liver, cholelithiasis, amyloidosis, and primary sclerosing cholangitis
• Rare extraintestinal manifestations (<1% of pediatric IBD patients): hematologic abnormalities, venous thrombosis, pancreatitis, nephrolithiasis, pulmonary disease, neurologic disease

Rabizadeh S, Oliva-Hemkey M: Extraintestinal manifestations of pediatric inflammatory bowel disease. In Mamula P, Markowitz J, Baldassano R, editors: Pediatric Inflammatory Bowel Disease, Springer, 2007, Philadelphia, pp 87-92.

151. What pharmacologic therapies are used in the treatment of ulcerative colitis and Crohn disease?
Mild disease and remission: 5-Aminosalicylic acids (ASA) (mesalamine, mesalazine), oral and rectal, particularly for ulcerative colitis; antibiotics; extended-release budesonide;
Moderate disease: Metronidazole (for Crohn disease); prednisone
Severe and refractory disease: Azathioprine; 6-Mercaptopurine; intravenous steroids; methotrexate; anti-tumor necrosis factor agents (e.g., infliximab, adalimumab); cyclosporine

Jacobstein D, Baldassano R: Inflammatory bowel disease. In Liacouras CA, Piccoli DA, editors: Pediatric Gastroenterology: The Requisites in Pediatrics, Philadelphia, 2008, Mosby, p 138.

152. How are therapies chosen for IBD?
Traditionally, medications have been chosen in a “step-up” approach using medications with less severe side effects such as ASA or steroids before initiation of immunomodulators or biologic therapies. This is now being challenged with a “top-down” approach using more potent medications earlier in the course of the disease to induce mucosal healing, interrupt the natural history of the disease processes, and decrease potential for long-term complications including need for surgical intervention. Overall, the age, presenting symptoms and severity, and sex of the child must be taken into consideration when starting therapy.
153. Is there a potential role for thalidomide in the treatment of Crohn disease? The use of thalidomide was one of the great medicinal tragedies of the twentieth century. Thalidomide was released initially in Germany in 1957 and marketed as a wonder drug for insomnia, anxiety, and nausea, especially morning sickness associated with pregnancy. Thousands of women who took the drug in the first trimester gave birth to infants with marked malformations of the extremities and other malformations. In 1998, it was resurrected as an anti-leprosy drug. In 2006, the Food and Drug Administration (FDA) approved a modified version of thalidomide for use in the treatment of multiple myeloma.
Thalidomide has properties of lowering tumor necrosis factor and inhibiting angiogenesis.
Investigators in Italy found that, compared with placebo, treatment with thalidomide for children and adolescents with refractory Crohn disease resulted in improved clinical remission at 8 weeks, which was maintained in a long-term period of continued treatment.

Lazzerini M, Martelossi S, Magazzu G, et al: Effect of thalidomide on clinical remission in children and adolescents with refractory Crohn disease: a randomized clinical trial, JAMA 310:2164–2173, 2013.

154. In a child who has been diagnosed with Crohn disease, what are potential long-term complications?
• Severe perianal disease can be a debilitating complication. More prevalent in patients with Crohn disease, it may range from simple skin tags to the development of perianal abscesses or fistulas.
• Enteroenteral fistulas may occur and “short circuit” the absorptive process. The thickened bowel may obstruct or perforate, thus requiring operation. The recurrence rate is high after surgery, repeated operations are often necessary, and short bowel syndrome may result.
In many cases, a permanent ostomy is placed, although pouch construction and continent ileostomies have become more common.
• Growth retardation and delayed puberty are seen extensively in patients with pediatric Crohn disease. The insidious onset may result in several years of linear growth failure before the correct diagnosis is made. With epiphyseal closure, linear growth is terminated, and short adult stature will be permanent.
• Decrease in bone mineralization (osteopenia) is a more commonly recognized complication of Crohn disease, secondary to growth failure and malnutrition, disease activity, and toxic effect
of corticosteroids. All patients should have a bone densitometry scan to assess for this complication. Treatment includes increased weight-bearing activity, correction of nutritional deficits, vitamin
D and calcium supplementation, and more aggressive medical treatment of disease.
• Hepatic complications of IBD include chronic active hepatitis and sclerosing cholangitis, which may require liver transplantation.
• Nephrolithiasis may occur in patients with resections or steatorrhea as a result of the increased intestinal absorption of oxalate.
• Chronic reactive and restrictive pulmonary disease has been noted.
• Arthralgias are common, but destructive joint disease is uncommon.

KEY POINTS: INFLAMMATORY BOWEL DISEASE
1. Ulcerative colitis is limited to the superficial mucosa of the large intestine, always involves the rectum, and demonstrates no skip lesions.
2. Crohn disease can occur anywhere in the gastrointestinal tract (from the mouth to the anus) and demonstrates transmural inflammation with skip lesions; noncaseating granulomas may be found on microscopic pathology. The transmural inflammation can result in the formation of abscesses or fistulas.
3. Potential long-term complications of inflammatory bowel disease include chronic growth failure, abscesses, fistulas, nephrolithiasis, and osteopenia.
4. Surgery can be curative for ulcerative colitis, but the incidence of postoperative recurrence is high in Crohn disease.

155. Are children with IBD at increased risk for malignancy? The risk for malignancy has not been studied systematically among pediatric populations with IBD. The risk in adults depends both on the disease and its duration. After 10 years of ulcerative colitis, the risk rises dramatically (1% to 2% increased incidence of malignancy per year). The risk is thought to be higher in patients with pancolitis compared with those with limited left-sided disease. The carcinomas associated with ulcerative colitis are often poorly differentiated and metastasize early; they have a poorer prognosis and are more difficult to identify by radiographic and colonoscopic examinations. Most authors indicate that carcinoma of the bowel is much less common among patients with Crohn disease, although this has been disputed. The risk for lymphoma is increased in patients with Crohn disease. Immunosuppressive
(e.g., 6-mercaptopurine) and biologic (e.g., infliximab) therapy may also increase the risk for neoplasia.
156. When is surgery indicated for children with IBD?
See Table 7-11.

NUTRITION
157. What are various requirements for protein, fat, and carbohydrates? Protein should account for 7% to 15% of caloric intake and should include a balance of the 11 essential amino acids. Protein requirements range from 0.7 to 2.5 g/kg per day. Fats should provide 30% to 50% of caloric intake. Although most of these calories are derived from long-chain triglycerides, sterols,

Table 7-11. Indications for Surgery for Children with IBD
CROHN DISEASE ULCERATIVE COLITIS
Perforation with abscess formation Urgent:
Obstruction with or without stenosis Hemorrhage
Uncontrolled massive bleeding Perforation
Draining fistulas and sinuses Toxic megacolon
Toxic megacolon Acute fulminant colitis unresponsive to maximal
Growth failure in patients with localized medical therapy areas of resectable disease
ELECTIVE:
Chronic disease with recurrent severe exacerbations Continuous incapacitating disease despite adequate
medical treatment
Growth retardation with pubertal delay
Disease of >10 years’ duration with evidence of epithelial dysplasia
From Hofley PM, Piccoli DA: Inflammatory bowel disease in children, Med Clin North Am 78:1293–1295, 1994.

medium-chain triglycerides, and fatty acids may be important in certain diets. Linoleic acid and arachidonic acid are essential for tissue membrane synthesis, and about 3% of intake must be composed of these triglycerides. The remaining 50% to 60% of calories should come from carbohydrates. About half of these are contributed by monosaccharides and disaccharides (e.g., sucrose, lactose) and the remainder by starches.

American Dietetic Association: www.eatright.org. Accessed on Mar. 20, 2015.

158. If recommended caloric intakes are maintained, what is normal daily weight gain of young children?
See Table 7-12.

Table 7-12. Normal Daily Weight Gain in Young Children*

AGE WEIGHT GAIN RECOMMENDED
(G) CALORIC INTAKE (KCAL/KG/DAY)
0-3 mo 26-31 100-120
3-6 mo 17-18 105-115
6-9 mo 12-13 100-105
9-12 mo 9 100-105
1-3 yr 7-9 100
4-6 yr 6 90
*It should be noted that, when babies are primarily breastfed, growth during months 3 to 18 is less than that indicated by the table. On average, breastfed babies gain 0.65 kg less than formula-fed infants during the first year of life.
Data from Dewey KG, Heinig MJ, Nommsen LA, et al: Growth of breast-fed and formula-fed infants from 0 to 18 months: the DARLING Study, Pediatrics 89:1035–1041, 1992; and National Research Council, Food and Nutrition Board: Recommended Daily Allowances. Washington, DC, 1989, National Academy of Sciences.

159. What are the recommended bottle feedings by age?
See Table 7-13.

Table 7-13. Recommended Bottle Feedings by Age

AGE NUMBER OF FEEDINGS FLUID OUNCES PER FEEDING
Birth-1 week 6-10 1-3
1 week-1 month 7-8 2-4
1-3 months 5-7 4-6
3-6 months 4-5 6-7
6-9 months 3-4 7-8

160. Why should whole cow milk not be introduced until 1 year of age?
Introduction of whole cow milk to infants <1 year of age is known to be detrimental to infants. It is associated with iron deficiency anemia because of its low iron content and occult intestinal blood loss, which occurs in 40% of normal young infants being fed cow milk. Early use of whole milk
may contribute to weight acceleration and the development of overweight/obesity.
161. Why is honey not recommended for infants during the first year of life? Honey has been associated with infantile botulism as have some commercial corn syrups. Clostridium botulinum spores contaminate the honey and are ingested. In infants, intestinal colonization and multiplication of the organism may result in toxin production and lead to symptoms of constipation, listlessness, and weakness.
162. How is nutritional status objectively assessed in children?
• Growth chart: Anthropometric data give an estimate of the height, weight, and head circumference of a child compared with a population standard. A change in the child’s percentile months may signify the presence of a nutritional problem or systemic disease.
• Compare actual with ideal body weight (average weight for height age): The ideal body weight is determined by plotting the child’s height on the 50th percentile and recording the corresponding age. The 50th percentile weight for that age is obtained, and this ideal body weight is divided by the actual weight. The result is expressed as a percentage—the percent ideal body weight—that gives a better stratification of patients with significant malnutrition. An ideal body weight percentage of more than 120% is obese, 110% to 120% is overweight, 90% to 110% is normal, 80% to 90% is mild wasting, 70% to 80% is moderate wasting, and less than 70% is severe wasting.
• Measurement of midarm circumference: This provides information about the subcutaneous fat stores, and the midarm-muscle circumference (calculated from the triceps skinfold thickness) estimates the somatic protein or muscle mass.
• Laboratory assessment: Vitamin and mineral status can be directly assayed. Measurements of albumin (half-life, 14 to 20 days), transferrin (half-life, 8 to 10 days), and prealbumin (half-life, 2 to 3 days) can provide information about protein synthesis, but each may be affected
by certain diseases. The ratio of albumin to globulin may decrease in patients with protein malnutrition.
163. What features on examination of the scalp, eyes, and mouth suggest problems of malnutrition?
See Table 7-14.
164. How do marasmus and kwashiorkor differ clinically?
• Kwashiorkor is edematous malnutrition as a result of low serum oncotic pressure. The low serum proteins result from a disproportionately low protein intake compared with the overall caloric intake. These children appear replete or fat, but they have dependent edema, hyperkeratosis, and atrophic hair and skin. They generally have severe anorexia, diarrhea, and frequent infections, and they may have cardiac failure.
• Marasmus is severe nonedematous malnutrition caused by a mixed deficiency of both protein and calories. Serum protein and albumin levels are usually normal, but there is a

Table 7-14. Effects of Malnutrition on Scalp, Eyes, and Mouth
CLINICAL SIGN NUTRIENT DEFICIENCY
Epithelial
Skin
Xerosis, dry scaling Essential fatty acids
Hyperkeratosis, plaques around hair follicles Vitamin A
Ecchymoses, petechiae Vitamin K
Hair
Easily plucked, dyspigmented, lackluster
Protein calorie
Mucosal
Mouth, lips, and tongue B vitamins
Angular stomatitis (inflammation at corners of the mouth) B2 (riboflavin)
Cheilosis (reddened lips with fissures at angles) B2, B6 (pyridoxine)
Glossitis (inflammation of tongue) B6, B3 (niacin), B2
Magenta tongue B2
Edema of tongue, tongue fissures B3
Spongy, bleeding gums Vitamin C
Ocular
Conjunctival pallor due to anemia

Bitot spots (grayish, yellow, or white foamy spots on the whites of the eyes)
Vitamin E (premature infants), iron, folic acid, B12, copper
Vitamin A

marked decrease in muscle mass and adipose tissue. Signs are similar to those noted in hypothyroid children, with cold intolerance, listlessness, thin sparse hair, dry skin with decreased turgor, and hypotonia. Diarrhea, anorexia, vomiting, and recurrent infections may be noted.

165. What vitamins and minerals are often deficient in strict vegans and some vegetarians?
Vitamin B12, iron, calcium, and zinc. The groups most at risk are infants, children, and pregnant and lactating women. Semivegetarian diets rarely lead to such deficiencies.
166. What two factors make vitamin D deficiency such a common problem?
Changes in sun exposure/use of sunscreen and increases in obesity. Very few foods naturally contain vitamin D or are fortified with vitamin D. Exceptions are cod liver, tuna, fortified milk, and orange juice. The major source of vitamin D has been exposure to natural sunlight. If an individual wears a sunscreen with a protection factor of 30 or more, vitamin synthesis in the skin is reduced
by >95%. If an individual has darker skin, which provides more natural sun protection, he or she requires 3 to 5 times longer exposure to make the same amount of vitamin D as a person with a
white skin tone. Obesity is also a risk factor, because fat sequesters vitamin D. As sun exposure is reduced because of concerns about potential future malignancies and as obesity rates remain high, vitamin D deficiency is likely to remain a problem.

Holick MF, Binkley NC, et al: Evaluation, treatment and prevention of vitamin D deficiency: an Endocrine Society clinical practice guideline, J Clin Endocrinol Metab 96:1911–1930, 2011.

167. How much vitamin D should children receive on a daily basis?
2015 recommendations are that children without risk factors should receive the following amounts of vitamin D at a minimum:
Infants (<1 year): 400 IU/day
Children (1 to 18 years): 600 IU/day

All exclusively breastfed infants should receive 400 IU/day of vitamin D supplement because breast milk is low in vitamin D. Some formula-fed infants also require supplementation if their intake is less than approximately 33 ounces of formula daily, which is the quantity needed to receive the recommended amount of vitamin D.
168. What are the cutoffs for vitamin D deficiency and sufficiency?
Levels <20 ng/mL are considered vitamin D deficient; a level >20 ng/mL is considered sufficient, but levels that are >30 ng/mL are more preferable.

OBESITY AND LIPID DISORDERS
169. What are the weight status categories for children in terms of body mass index (BMI) percentile?
• Underweight: <5th percentile
• Healthy weight: 5th to 85th percentile
• Overweight: 85th to 95th percentile,
• Obese: 95th to 98th percentile
• Severely (morbidly) obese: ≤99th percentile
170. What screening laboratory tests should be done for obese children?
For children who are obese or severely obese, a basic laboratory evaluation to rule out the presence of obesity-related metabolic abnormalities should be considered. These would include:
• Liver function tests (AST and ALT): to assess possible NAFLD
• Fasting lipid profile: elevated triglycerides and reduced HDL is highly suggestive of significant insulin resistance
• CBC: iron deficiency and iron deficiency anemia are common in obese children
• Fasting glucose: sensitivity, however, is low to detect glucose intolerance. A standard oral glucose tolerance test should be considered for severe obesity, positive family history of type 2 diabetes or when acanthosis nigricans is present.
• Vitamin D level: deficiency is common in obese children.
• Thyroid function tests
Other tests should be determined if comorbidities are suspected by history or exam.

Baker JL, Farpour-Lambert NJ, Nowicka A, et al: Evaluation of the overweight/obese child—practical tips for the primary health care provider: recommendations from the Childhood Obesity Task Force of the European Association for the Study of Obesity, Obes Facts 3:131–137, 2010.

171. What are the different types of cholesterol?
Triglycerides: the major form of fat in the body
LDL: low density lipoprotein; the “bad” cholesterol; formed from VLDL or chylomicrons; saturated and trans fats increase LDL; major carrier of cholesterol into the body tissues
HDL: high density lipoprotein; “good” cholesterol; synthesized in the liver and gut; major carrier of cholesterol away from the body tissues
VLDL: very low density lipoprotein; made by the liver; high in triglycerides
Chylomicrons: transports dietary fat from intestines to liver and adipose tissues; high in triglycerides
Non-HDL (total cholesterol – HDL): can be used if a nonfasting lipid profile is obtained or if triglycerides are >400

172. Why is the promotion of cardiovascular health and the identification of specific risk factors important in pediatric medicine?
• Atherosclerotic changes originate in childhood.
• Risk factors for the development of atherosclerosis can be identified in childhood.
• The progression of atherosclerosis relates to the number and intensity of these risk factors.
• Risk factors track from childhood to adult years.
• Interventions exist for the management of identified risk factors.

Expert Panel on Integrated Guidelines: Expert panel on integrated guidelines for cardiovascular health and risk reduction in children and adolescents: summary report, Pediatrics 128: S217, 2011.

173. What are the screening guidelines for lipids?
Evidence-based guidelines from an expert panel from the National Heart Lung and Blood Institute (NHLBI) include the following:

Birth to 2 years: No lipid screening is recommended.
2 to 8 years: No universal screening is recommended unless there are risk factors for cardiovascular disease (see below)
9 to 11 years: Universal screening with nonfasting lipid panel is recommended. Obtain fasting lipid panel twice and average the results if non-HDL 145 mg/dL or HDL <40, or check fasting lipid panel and repeat if LDL 130 mg/dL, non-HDL 145 mg/dL, HDL <40, or triglycerides 100 mg/dL (if <10 years) or 130 mg/dL (if >10 years).
12 to 16 years: No routine screening is recommended unless there are risk factors for
cardiovascular disease (see below). Universal lipid screening is not recommended in this age group because of decreased sensitivity and specificity for predicting adult values, particularly LDL.
17 to 19 years: Universal screening is recommended once in this age group with a nonfasting or fasting lipid profile; repeat in 2 weeks to 3 months if abnormal.
Risk factors: parent with total cholesterol 240 mg/dL, early heart disease in a first or second degree RELATIVE, diabetes (type I or II), hypertension, BMI 95th percentile, smokes cigarettes, kidney disease, heart disease, chronic inflammatory disease, or HIV infection

Expert Panel on Integrated Guidelines: Expert panel on integrated guidelines for cardiovascular health and risk reduction in children and adolescents: summary report, Pediatrics 128: S239, 2011.

174. What are the cutoffs for abnormal lipid levels?
See Table 7-15.

Table 7-15. Lipid Levels in Children and Adolescents

CATEGORY ACCEPTABLE MG/DL BORDERLINE MG/DL HIGH MG/DL LOW MG/DL
TC <170 170-199 > 200
LDL-c <110 110-129 > 130
Non HDL-c <120 120-144 > 145
TG
0-9 yr <75 75-99 > 100
10-19 yr <90 90-129 > 130
HDL-c >45 40-45 <40
HDL-c ¼ High-density lipoprotein cholesterol; LDL-c ¼ Low-density lipoprotein cholesterol; TC ¼ total cholesterol; TG ¼ triglycerides.
175. What are the American Heart Association dietary strategies for all children older than 2 years?
• Balance dietary calories with physical activity to maintain normal growth.
• Engage in 60 minutes of moderate to vigorous play or physical activity daily.
• Eat vegetables and fruit daily, limit juice intake.
• Use vegetable oils and soft margarines low in saturated fat and trans fatty acids instead of butter or most other animal fats in the diet.
• Eat whole-grain breads and cereals rather than refined-grain products.
• Reduce the intake of sugar-sweetened beverages and foods.
• Use nonfat (skim) or low-fat milk and dairy products daily.
• Eat more fish, especially oily fish, broiled or baked.
• Reduce salt intake, including salt from processed foods.

American Heart Association: Dietary recommendations for children and adolescents: a guide for practitioners, Pediatrics
117:544–559, 2006.

176. How are the primary genetic hyperlipidemias classified?
See Table 7-16.

Table 7-16. Classification of Primary Genetic Hyperlipidemias
FREDERICKSON TYPE LIPIDS INCREASED LIPOPROTEINS INCREASED
PREVALENCE CLINICAL FINDINGS
I Triglyceride Chylomicrons Very rare Eruptive xanthomas,
pancreatitis,
recurrent abdominal
pain, lipemia
retinalis,
hepatosplenomegaly
IIa Cholesterol LDL Common Tendon xanthomas, PVD
IIb Cholesterol, triglyceride LDL+ VLDL Common PVD, no xanthomas
III Cholesterol, triglyceride VLDL remnants (IDL) Rare PVD, yellow palm creases
IV Triglyceride VLDL Uncommon PVD, xanthomas, hyperglycemia
V Triglyceride, cholesterol VLDL
+ chylomicrons Very rare Pancreatitis, lipemia retinalis, xanthomas, hyperglycemia
IDL ¼ Intermediate-density lipoprotein; LDL ¼ low-density lipoprotein; PVD ¼ peripheral vascular disease; VLDL ¼ very-low-density lipoprotein.

177. What is the most common hyperlipidemia in childhood?
Familial hypercholesterolemia, type IIA, with elevated cholesterol and LDL. This condition results from a lack of functional LDL receptors on cell membranes as a result of various mutations. When LDL cannot attach and release cholesterol to the cell, feedback suppression of hydroxymethylglutaryl coenzyme A reductase (the rate-limiting enzyme of cholesterol synthesis) does not occur, and cholesterol synthesis continues excessively. In the homozygous form of type IIa, xanthomas may appear before the age of 10 years and vascular disease before the age of 20 years. However, the homozygous form is very rare, with an incidence of 1 in 1,000,000 births. The heterozygous variety has a much higher incidence of 1 in 500, but it is less likely to produce clinical manifestations in children.

SURGICAL ISSUES
178. What is the natural history of an umbilical hernia?
Most umbilical hernias smaller than 0.5 cm spontaneously close before a patient is 2 years old. Those between 0.5 and 1.5 cm take up to 4 years to close. If the umbilical hernia is larger than 2 cm, it may still close spontaneously, but may take up to 6 years or more to do so. Unlike an inguinal hernia, incarceration and strangulation are rare with an umbilical hernia.

Barreto L, Khan AR, Khanbhal M, et al: Umbilical hernia, BMJ 347:f4252, 2013. Yazbeck S: Abdominal wall developmental defects and omphalomesenteric remnants. In Roy CC, editor: Pediatric Clinical Gastroenterology, ed 4. St. Louis, 1995, Mosby-Year Book, pp 134–135.

179. Which umbilical hernias warrant surgical repair?
Because of the high probability of self-resolution, indications for surgery are controversial. Some authorities argue that a hernia larger than 1.5 cm at the age of 2 years warrants closure as a result of its likely persistence for years. Others argue that, because the likelihood of incarceration is small for

umbilical hernias, surgical closure is warranted before puberty only for persistent pain, history of incarceration, or associated psychological disturbances.
180. When should an infant with inguinal hernia have it electively repaired? After the diagnosis of inguinal hernia is made, it should be repaired as soon as possible. In one large study of children with incarcerated hernia, 40% of patients had a known inguinal hernia before incarceration, and 80% were awaiting elective repair. Eighty percent of the children with incarceration of a hernia were infants younger than 1 year. Delay of repair should be minimized, especially in this age group. Another study found that if an infant presents with an incarcerated hernia, subsequently reduced in the ED, the potential for recurrent incarceration during a waiting period is increased 12-fold.

Chen LE, Zamakhshary M, Foglia RP, et al: Impact of wait time on outcome for inguinal hernia repair in infants, Pediatr Surg Int 25:225–232, 2009.
Stylianos S, Jacir NN, Harris BH: Incarceration of inguinal hernia in infants prior to elective repair, J Pediatr Surg 18:582– 583, 1993.

181. Does surgical repair of one hernia warrant intraoperative exploration for another on the opposite side?
This is a controversial topic. Many surgeons opt to have pediatric patients undergo exploration of the contralateral side during a hernia repair because 60% of infants will have a patent processus vaginalis on the opposite side. By age 2 years, approximately 10% of these become clinical hernias, although a large percentage do spontaneously obliterate before that time. Other surgeons feel the potential risk of contralateral exploration (e.g., injury to the vas deferens, testes, and ilioinguinal nerve) mandates a watchful waiting approach. Surveys of pediatric surgeons indicate persistent widespread practice variability.

Palmer LS: Hernias and hydroceles, Pediatr REV 34:457–463, 2013.
Ron O, Eaton S, Pierro A: Systematic review of the risk of developing a metachronous contralateral inguinal hernia in children, Br J Surg 94:804–811, 2007.

182. How are incarcerated inguinal hernias reduced?
Incarceration occurs most commonly during the first year of life. About one-third of infants
<2 months of age with inguinal hernias will develop incarceration. Because the infant will likely need to be admitted, nothing should be given to eat or drink. Reduction is most easily
accomplished if the infant is calm (preferably asleep), warm, and, if possible, in a slightly reverse Trendelenburg position. Analgesia (e.g., 0.1 mg/kg of intravenous morphine) may facilitate the relaxed state. With one hand, the examiner stabilizes the base of the hernia by the internal inguinal ring and, with the other hand, milks the sac distally to progressively force fluids and/or gas through the ring to eventually allow complete reduction. If unsuccessful, immediate surgery is indicated.
183. Under what clinical settings should manual reduction of an inguinal hernia not be attempted?
Reduction should not be attempted if the patient has clinical findings of shock, perforation, peritonitis, GI bleeding or obstruction, or evidence of gangrenous bowel (bluish discoloration of the abdominal wall).
184. What is the significance of green vomiting during the first 72 hours of life? During the neonatal period, green vomiting should always be interpreted as a sign of potential intestinal obstruction potentially requiring surgical intervention. In one study of 45 infants with green vomiting, 20% had surgical conditions (e.g., malrotation, jejunal atresia, jejunal stenosis), 10% had nonsurgical obstruction (e.g., meconium plug, microcolon), and 70% had idiopathic
vomiting that self-resolved. Plain radiographs frequently can be normal, particularly for malrotation, and thus falsely reassuring.

Williams H: Green for danger! Intestinal malrotation and volvulus, Arch Dis Child Educ Pract Ed 92:ep87–ep97, 2007. Lilien LD, Srinivasan G, Pyati SP, et al: Green vomiting in the first 72 hours in normal infants, Am J Dis Child 140:662–664, 1986.

185. What are the clinical findings of malrotation of the intestine? The lesion may display in utero volvulus, or it may be asymptomatic throughout life. Infants may display intermittent vomiting or exhibit signs compatible with complete obstruction. Any infant with bilious vomiting should be considered emergent and requires careful evaluation for volvulus and other high- grade surgical obstructions. Recurrent abdominal pain, distention, or lower GI bleeding may result from intermittent volvulus. Full volvulus with arterial compromise results in intestinal necrosis, peritonitis, perforation, and an extremely high incidence of mortality. Because of the extensive nature of the lesion, postoperative short gut syndrome is present in many patients who require resection. An upper
GI contrast study is the examination of choice when the diagnosis is suspected.
186. What causes the intestinal obstruction in malrotation?
Malrotation of the intestine is the result of the abnormal rotation of the intestine in the tenth week
of gestation around the superior mesenteric artery. Arrest of this counterclockwise rotation may occur at any degree of rotation. The cecum is unattached and located in the upper abdomen. One consequence of improper fixation of the mesentery allows for twisting (volvulus). Additionally, abnormal tissue (Ladd bands) connects the abnormally located cecum to the abdominal wall and may create a duodenal blockage (Fig. 7-11).

Figure 7-11. Incomplete intestinal rotation. Ladd bands are seen attaching the cecum to the right posterior abdominal wall. The duodenum can become compressed and possibly obstructed. (From Holcombe GW, Murphy JP, Ostlie DJ: Ashcraft’s Pediatric Surgery, ed 6.
Philadelphia, 2014, ELSEVIER, pp 430–438.)

187. In an asymptomatic child with an incidental finding of malrotation, is surgery indicated?
Because of the persistent possibility of acute volvulus and intestinal obstruction, surgery is always indicated when intestinal malrotation is diagnosed.

188. What is the most common cause of intestinal obstruction in young children? Intussusception, which occurs when one portion of the bowel invaginates into the other is the most common cause of intestinal obstruction in young children. Intussusception usually occurs before the second year of life; half of all cases occur between the ages of 3 and 9 months.
189. In what settings should intussusception be suspected?
Colicky pain is seen in more than 80% of cases, but it may be absent. It typically lasts 15 to 30 minutes, and the baby usually sleeps between attacks. In about two-thirds of cases, there is blood in the stool (currant jelly stools). Other presenting symptoms include massive lower GI bleeding or blood streaking on the stools. The infant may appear quite toxic, dehydrated, or in
shock; fever and tachycardia are common. A right lower quadrant mass may be palpable, or the area may feel surprisingly empty. Distention may accompany decreased bowel sounds.
190. How commonly does intussusception appear with the classic findings?
The classic triad of intussusception (colicky pain, vomiting, and passage of bloody stool) is the exception; overall, 80% of patients do not have this triad of symptoms. About 30% have blood in the stool, and this percentage may drop to about 15% if the abdominal pain was present for less than 12 hours. Palpation of a mass can suggest the diagnosis, but generally a high degree of suspicion is important. Delay in diagnosis is common.

Lochhead A, Jamjoom R, Ratnapalan S: Intussusception in children presenting to the emergency department,
Clin Pediatr 52:1029–1033, 2013.
Klein EJ, Kapoor D, Shugerman RP: The diagnosis of intussusception, Clin Pediatr 43:343–347, 2004.

191. What causes intussusception?
Intussusception is caused by one proximal segment of the bowel being invaginated and progressively drawn caudad and encased by the lumen of distal bowel. This causes obstruction and may occlude the vascular supply of the bowel segment. There is commonly a lead point on the proximal bowel that initiates the process. Lead points have included juvenile polyps, lymphoid hyperplasia, hypertrophied Peyer patches, eosinophilic granuloma of the ileum, lymphoma, lymphosarcoma, leiomyosarcoma, leukemic infiltrate, duplication cysts, ectopic pancreas, Meckel diverticulum, hematoma, Henoch-Sch€onlein syndrome, worms, foreign bodies, and appendicitis.

Waseem M, Rosenberg HK: Intussusception, Pediatr Emerg Care 24:793–800, 2008.

192. What is the most common type of intussusception?
Ileocolic intussusception (Fig. 7-12) is most common, and it is also the most common cause
of intestinal obstruction during infancy. Cecocecal and colocolic intussusceptions are less common. Gastroduodenal intussusception is rare and is usually associated with a gastric mass lesion such as a polyp or a leiomyoma. Enteroenteral intussusception is seen after surgery and in patients
with Henoch-Sch€onlein syndrome.

Figure 7-12. Intraoperative appearance of ileocolic intussusception through the
ileocecal valve. (From Wyllie R, Hyams JS, Kay M, editors: Pediatric Gastrointestinal and Liver Disease, ed 3. Philadelphia, 2006, Saunders,
p 717.)

193. How is intussusception diagnosed?
Radiographs can demonstrate a small bowel obstruction pattern, but the sensitivity is low (45%), so this is not typically used to diagnose intussusception. Ultrasound is increasingly being used to make this diagnosis and has a role in the evaluation of reducibility, potential pathologic lead point, and exclusion of residual intussusception after enema. Traditionally, the diagnostic study of choice is a barium enema because this can be both diagnostic and therapeutic. Air enema is now considered to be better at reduction, safer, faster, and result in less radiation compared with barium enemas. In 74% of cases, air enema under fixed hydrostatic pressure will reduce the intussusception. If this is unsuccessful, surgical reduction is necessary.

Applegate KE: Intussusception in children: evidence-based diagnosis and treatment, Pediatr Radiol 39: S140–S143, 2009.

194. How frequently does intussusception recur?
Overall recurrence rates for intussusception are about 13%. The recurrence rate during the first 24 hours is low, 2% to 4%, so the vast majority of recurrences will not be identified by overnight hospitalization.

Gray MP, Li S-H, Hoffman R, et al: Recurrence rates after intussusception enema reduction: a meta-analysis,
Pediatrics 134:110–119, 2014.

195. Rotavirus vaccine and intussusception: how are they intertwined?
Rotashield, an oral rotavirus vaccine licensed in the United States in 1998, was suspended from use in 1999 when increased rates of intussusception were noted. Two new rotavirus vaccines, RotaTeq and Rotarix, were licensed in 2006 and 2008, respectively. International postlicensure studies and U.S. data have demonstrated a slightly increased risk for intussusception during the first 3 weeks after the first dose of both vaccines.

Yih WK, Lieu TA, Kulldorff M, et al: Intussusception risk after rotavirus vaccination in U.S. infants, N Engl J Med
370:503–512, 2014.

196. Duodenal or jejunoileal atresia: which is associated with other embryonic abnormalities?
Duodenal atresia. Duodenal atresia is caused by a persistence of the proliferative stage of gut development and a lack of secondary vacuolization and recanalization. It is associated with a high incidence of other early embryonic abnormalities. Extraintestinal anomalies occur in two-thirds
of patients with this condition.
Jejunoileal atresia occurs after the establishment of continuity and patency as evidenced by distal meconium seen in these patients. The etiology is postulated to be a vascular accident, volvulus, or mechanical perforation. Jejunoileal atresias are usually not associated with any other systemic abnormality.

197. What is the classic radiographic finding in duodenal atresia?
The double bubble. Swallowed air distends the stomach and the proximal duodenum (Fig. 7-13).

198. How does the infant with biliary atresia classically present?
Biliary atresia is a condition in which the extrahepatic biliary system is obliterated, and bile flow is obstructed. In classic cases, an otherwise healthy-appearing term infant develops a recognizable jaundice by the third week of life, with increasingly dark urine and acholic stools. Usually, the child appears well, with acceptable growth. The skin color sometimes appears somewhat greenish yellow. The spleen becomes palpable after the third or fourth week, at which time the liver is usually hard and enlarged. In other cases, the jaundice is clearly present in the conjugated form during the first week of life. There is also a strong association between the polysplenia syndrome and earlier presentation of biliary atresia.

Figure 7-13. Duodenal atresia. (From Zitelli BJ, DAVIS HW: Atlas of Pediatric Physical Diagnosis, ed 5. Philadelphia, 2007, Mosby, p 637.)

199. What is the surgical procedure for biliary atresia?
The Kasai procedure (hepatoportoenterostomy). The remnants of the extrahepatic biliary tree are identified, and a cholangiogram is performed to verify the diagnosis. An intestinal limb (Roux-en-Y) is attached to drain bile from the porta hepatis.
200. Which is accompanied by more complications: high or low imperforate anus? High-type imperforations. The distinction is based on whether the blind end of the terminal bowel or rectum ends above (high type) or below (low type) the level of the pelvic levator musculature. The patients with high-type imperforations will have ectopic fistulas (rectourinary, rectovaginal), urologic anomalies (hydronephrosis or double collecting system), and lumbosacral spine defects (sacral agenesis, hemivertebrae). The surgical repair in these patients is much more extensive, and future problems of incontinence, fecal impaction, and strictures are
more likely.
201. What is the classic presentation of pyloric stenosis?
An infant 3 to 6 weeks old has progressive nonbilious projectile vomiting leading to dehydration with hypochloremic, hypokalemic, metabolic alkalosis. On physical examination, a pyloric “olive” is palpable, and peristaltic waves are visible.
202. How is pyloric stenosis diagnosed?
If the classic signs and symptoms are present in association with the typical blood chemistry findings (hypochloremia, hypokalemia, metabolic alkalosis) and a mass is palpated, the diagnosis can be made on clinical grounds. If the diagnosis is in doubt, ultrasound can be used to visualize the hypertrophic pyloric musculature (Fig. 7-14). Upper GI contrast studies demonstrate pyloric obstruction with the characteristic “string sign” and enlarged “shoulders” bordering the elongated and obstructed pyloric channel.
203. What is the mechanism of hyperbilirubinemia in babies with pyloric stenosis? Unconjugated hyperbilirubinemia has been noted in 10% to 25% of babies with pyloric stenosis. Although an enhanced enterohepatic circulation for bilirubin probably plays a role in the pathogenesis of the hyperbilirubinemia, hepatic glucuronyl transferase activity is markedly depressed in these jaundiced infants. The mechanism of diminished glucuronyl transferase activity is not known, although inhibition of the enzyme by intestinal hormones has been suggested.
204. In a patient with suspected pyloric stenosis, why is an acidic urine very worrisome?
As vomiting progresses in infants with pyloric stenosis, a worsening hypochloremic metabolic alkalosis develops. Multiple factors (e.g., volume depletion, elevated aldosterone levels) result

Figure 7-14. A, Ultrasound of pyloric stenosis. Note the elongated and curved pyloric channel with parallel walls and the thickened muscle with a “shoulder” projecting into the antrum. B, Longitudinal sonograph of the pylorus in a patient with pyloric stenosis. 1, canal length 1.7 cm; 2, muscle wall thickness 0.6 cm. (From Glick PL, Pearl RH, Irish MS, Caty MG: Pediatric Surgery Secrets. Philadelphia, 2001, Hanley & Belfus, p 203.)

in maximal renal efforts to reabsorb sodium. In the distal tubule, this is typically achieved by exchanging sodium for potassium and hydrogen. When total-body potassium levels are very low, hydrogen is preferentially exchanged, and a paradoxic aciduria develops (in the setting of an alkaline plasma). This acidic urine is an indication that intravascular volume expansion and electrolyte replenishment (especially chloride and potassium) are urgently needed.
205. What is the connection between pyloric stenosis and macrolide antibiotics?
The use of erythromycin during the first 2 weeks after birth is associated with an increased risk, up to 30-fold, for the development of pyloric stenosis. Azithromycin use increases the risk up to 8-fold. Use of erythromycin or azithromycin from 2 weeks to 4 months of age is also associated with an increased risk, albeit smaller. Speculation on a mechanism involves possible macrolide effects as a prokinetic agent on gastrointestinal smooth muscle, which could cause spasm of the pyloric muscle.

Eberly MD, Eide MB, Thompson JL, Nylund CM: Azithromycin in early infancy and pyloric stenosis, Pediatrics 135:483– 488, 2015.
Lund M, Pasternak B, Davidsen RB, et al: Use of macrolides in mother and child and risk of infantile hypertrophic pyloric stenosis: nationwide cohort study, BMJ 348:1908–1918, 2014.

206. What is the short bowel syndrome?
The short bowel syndrome results from extensive resection of the small intestine. Normally, most carbohydrates, proteins, fats, and vitamins are absorbed in the jejunum and the proximal ileum. The terminal ileum is responsible for the uptake of bile acids and vitamin B12. Short bowel syndrome results in FTT, malabsorption, diarrhea, vitamin deficiency, bacterial contamination, and gastric hypersecretion.
207. Why are infants with short bowel syndrome prone to renal calculi?
Chronic intestinal malabsorption results in an increase of intraluminal fatty acids, which saponify with dietary calcium. Thus, nonabsorbable calcium oxalate does not form, excessive oxalate is absorbed, and hyperoxaluria with crystal formation results.
208. In extensive small bowel resection, how much is “too much”?
Infants who retain 20 cm of small bowel as measured from the ligament of Treitz can survive if the ileocecal valve is intact. If the ileocecal valve has been removed, the infant usually requires a minimum of 40 cm of bowel to survive. The importance of the ileocecal valve appears to relate to its ability to retard transit time and minimize bacterial contamination of the small intestine.

KEY POINTS: SURGICAL ISSUES
1. Bilious (dark green) emesis in a newborn is a true gastrointestinal emergency; it is a sign of potential obstruction.
2. The classic triad of intussusception consists of the following: (1) colicky abdominal pain, (2) vomiting, and (3) bloody stools with mucus. However, it occurs in fewer than 20% of patients.
3. In patients younger than 2 years, intussusception is the most common abdominal emergency.
4. Pyloric stenosis typically presents with progressive, nonbilious, projectile vomiting and a hypochloremic, hypokalemic metabolic alkalosis in an infant between the ages of 3 and 6 weeks.
5. The classic picture of appendicitis is anorexia followed by pain followed by nausea and vomiting, with subsequent localization of findings to the right lower quadrant. However, there is a large degree of variability, particularly in younger patients.

209. Appendicitis in children: clinical, laboratory, or radiologic diagnosis?
The diagnosis of appendicitis has traditionally been a clinical one. The classic picture in children is a period of anorexia followed by pain, nausea, and vomiting. Abdominal pain begins periumbilically and then shifts after 4 to 6 hours to the right lower quadrant. Fever is low grade. Peritoneal signs are detected on examination. In unequivocal cases, experienced surgeons would argue that no laboratory tests are needed.
Laboratory studies have limited value in equivocal cases. White blood cell count of more than 18,000/mm3 or a marked left shift is unusual in uncomplicated cases and suggests perforation or another diagnosis. A urinalysis with many white blood cells suggests a urinary tract infection as the primary pathology.
It is uncommon for children to undergo surgery for suspected appendicitis without imaging. CT has been considered the gold standard for diagnosis. Ultrasound has been studied as an alternative, but it is highly operator dependent with a wide reported sensitivity range.
Magnetic resonance imaging (MRI) has been used in combination with ultrasound for the diagnosis of appendicitis in adults, but studies of its use in pediatric medicine are limited.
The advantages of CT are less operator dependence; easier visualization of retrocecal appendix; and less interference with bowel gas, obesity, or the patient’s pain. A major concern is the extent of radiation exposure, which is considerable particularly with the use of contrast studies.

Aspelund G, Fingeret A, Gross E, et al: Ultrasonography/MRI versus CT for diagnosing appendicitis, Pediatrics
133:1–8, 2014.
Acheson J, Banerjee J: Management of suspected appendicitis in children, Arch Dis Child Educ Pract ED 95:9–13, 2010.

210. How specific is the diagnosis of appendicitis if an appendicolith is noted on radiograph?
Although an appendicolith (or fecalith) on radiographic studies (plain film or CT scan) is significantly associated with appendicitis, it is not sufficiently specific to be the sole basis for the diagnosis. On CT scanning, these can be noted in 65% of patients with appendicitis and in up to 15% of patients without appendicitis. The positive-predictive value of finding an appendicolith is about 75%; in its absence, the negative-predictive value is only 26%.

Lowe LH, Penney MW, Scheker LE, et al: Appendicolith revealed on CT in children with suspected appendicitis: How specific is it in the diagnosis of appendicitis? AJR Am J Roentgenol 175:981–984, 2000.

211. Should a digital rectal examination be performed on all children with possible appendicitis?
Tradition says yes, but reviews of studies of the practice indicate that in children it can be emotionally and physically traumatic and associated with a high false-positive interpretation.
It may be most helpful in equivocal cases involving pelvic or retrocecal appendicitis (about one- third of cases), suspected abscess formation, or for attempted palpation of adnexal or cervical tissues when vaginal examination is not indicated. Thus, many clinicians now view it as “investigatory” rather than “routine” and only when results will change management.

Brewster GS, Herbert ME: Medical myth: a digital rectal examination should be performed on all individuals with possible appendicitis, West J Med 173:207–208, 2000.

212. In children taken to surgery for suspected appendicitis, how often is perforation of the appendix present?
It depends to a large extent on the age of the child (and, of course, on the skill of the clinician).
Unfortunately, as a result of the variable location of the appendix, the clinical presentation of pain
in appendicitis is often very different from the classic case. The younger the child, the more difficult the diagnosis. In infants younger than 1 year, nearly 100% of patients who come to surgery have a perforation. Fortunately, appendicitis is rare in this age group because the appendiceal opening at the cecum is much larger than the tip, and obstruction is unusual. In children younger than 2 years, 70% to 80% are perforated; in those 5 years and younger, 50% are perforated. Particularly in younger children, a high index of suspicion is necessary, and rapid diagnosis is critical. If the onset of symptoms can be pinpointed (usually anorexia related to a meal), 10% of patients will have perforation during the first 24 hours, but more than 50% will perforate by 48 hours.
213. Should children with acute abdominal pain be given analgesia before a diagnosis?
A controversial question because a long-held fear has been that treating the pain may mask the symptoms, change the physical findings, and potentially delay the diagnosis of a possible surgical problem. However, there is growing evidence that the use of opiate analgesia in patients, including children, with acute abdominal pain does not result in increased mortality or morbidity.

Bailey B, Bergeron S, Gravel J, et al: Efficacy and impact of intravenous morphine before surgical consultation in children with right lower quadrant pain suggestive of appendicitis: a randomized controlled trial, J Pediatr 50:371–378, 2007.
Ranji SR, Goldman LE, Simel DL, et al: Do opiates affect the clinical evaluation of patients with acute abdominal pain? JAMA 296:1764–1774, 2006.

Acknowledgment
The editors gratefully acknowledge contributions by Drs. Douglas Jacobstein, Peter Mamula, Jonathan E. Markowitz, and David A. Piccoli that were retained from previous editions of Pediatric Secrets.

BONUS QUESTIONS
214. In what clinical settings is rectal prolapse most commonly seen?
• Constipation
• Celiac disease
• Malnutrition
• Severe coughing (e.g., pertussis)
• Cystic fibrosis
• Enterobius VERMICULARIS (pinworm) infestation
• Colonic polyps
• Myelomeningocele
• Abnormalities of sacrum or coccyx
• Ehlers-Danlos syndrome

215. List the indications for lower gastrointestinal colonoscopy or endoscopy in children.
• Hematochezia in the absence of an anal source
• History of familial polyposis
• Chronic diarrhea of unclear etiology
• Persistent severe unexplained mid to lower abdominal pain
• Colitis of unclear etiology
• Diagnosis and management of inflammatory bowel disease
• Removal of foreign body
• Ureterosigmoidostomy, surveillance
• Abnormality on barium enema
• Dilation of a colonic stricture

216. How can you tell a coin from a button battery on x-ray?
Button batteries typically have a “halo” sign when seen en face and have a step off (near the rim) when seen laterally. Coins are uniform. If the identity of a round foreign body cannot be reconciled by history between coin and button battery, it should be assumed to be a battery and managed more urgently or emergently.

217. What are the indications for nonemergent foreign body removal?
Asymptomatic objects that remain in the stomach for >4 weeks are unlikely to pass and should be removed. Coins often fall into this category. Children who ingest a coin(s), and who are asymptomatic and have an x-ray that reveals the coin to be in the stomach can often be observed. If the object has
not been retrieved in the stool, x-rays of metallic objects can be taken weekly to determine if the object has passed beyond the stomach. After 4 weeks, if the object remains in the stomach, it should be removed. Again, patients who report ANY symptoms (vomiting, pain, fever, dysphagia), at any time require emergent evaluation.
218. What are risk factors for the formation of phytobezoars?
Phytobezoars are trapped masses in the stomach and GI tract; they contain indigestible plant material (e.g., skins, fibers). Poor mastication, use of H2 blockers, gastroparesis, excessive intake of fibers, and a history of gastric surgery are risk factors for the development of phytobezoars. As Roux-en-Y gastric bypass has become more common, the incidence of phytobezoars has increased. As weight loss surgery becomes more common in the pediatric population, the incidence of phytobezoars may increase.
219. What tests, in addition to those ruling out Hirschsprung disease, are commonly considered for refractory cases of constipation?
Laboratory testing will include serum calcium, thyroid-stimulating hormone, lead, and celiac disease panel. A sweat test might be considered. A barium enema can evaluate for colonic anatomic abnormalities. Magnetic resonance imaging of the spine to rule out spina bifida occulta or tethered cord is sometimes helpful. Anorectal manometry can help diagnose a motility disorder.

220. Which children should be seen for the medical evaluation of acute diarrhea?
• Young age (<6 months or weighing <8 kg)
• History of premature birth, chronic medical conditions, or concurrent illness
• Fever 38 °C for infants <3 months old or 39 °C for children 3 to 36 months old
• Visible blood in stool
• High output, including frequent and substantial volumes of diarrhea
• Persistent vomiting
• Caregiver’s report of signs that are consistent with dehydration
• Change in mental status
• Suboptimal response to oral rehydration therapy or inability of caregiver to administer this therapy

King CK, Glass R, Bresee JS, Duggan C, Centers for Disease Control and Prevention: Managing acute gastroenteritis among children: oral rehydration, maintenance, and nutritional therapy, MMWR Recomm Rep 52:1–16, 2003.

221. What is the differential diagnosis of chronic diarrhea in different age groups?
• Newborn: Congenital short gut, congenital lactose intolerance, malrotation with intermittent volvulus, ischemia, defective sodium-hydrogen exchange, congenital chloride diarrhea, microvillus disease
• Infant: Protein sensitization, infection, parenteral diarrhea (during urinary or upper respiratory infection), immunoglobulin deficiency, diarrhea after gastroenteritis, cystic fibrosis, celiac disease, C. difficile infection
• Toddler: Diarrhea after gastroenteritis, food allergy, excessive ingestion of fruit juice, toddler’s diarrhea, hyperthyroidism, sucrase-isomaltase deficiency, constipation and impaction with overflow, teething
• Older children: Lactose intolerance, infection, inflammatory bowel disease, irritable bowel syndrome, laxative abuse

Keating JP: Chronic diarrhea, Pediatr REV 26:5–14, 2005.
Gryboski J: The child with chronic diarrhea, Contemp Pediatr 10:71–97, 1993.

222. What is the physiologic basis for oral rehydration therapy?
Intestinal solute transport mechanisms generate osmotic gradients by the movement of electrolytes and nutrients through the cell, and water passively follows. A coupled transport of sodium and glucose occurs at the intestinal brush border, and this is facilitated by the protein sodium glucose cotransporter 1. Oral replacement solutions are formulated with sufficient sodium, glucose, and osmolarity to maximize this cotransportation and to avoid problems of excessive sodium intake or additional osmotic diarrhea.

King CK, Glass R, Bresee JS, Duggan C, Centers for Disease Control and Prevention: Managing acute gastroenteritis among children: oral rehydration, maintenance, and nutritional therapy, MMWR Recomm Rep 52:1–16, 2003.

223. What are the basic principles guiding optimal treatment of children with diarrhea and mild dehydration?
• Oral rehydration solution should be used for rehydration.
• Oral rehydration should be performed rapidly, ideally 50 to 100 mL/kg over 3 to 4 hours.
• For rapid realimentation, an age-appropriate, unrestricted diet is recommended as soon as dehydration is corrected.
• For breastfed infants, nursing should be continued.
• For formula-fed infants, diluted formula is not recommended, and special formula is usually not necessary.
• Additional oral rehydration solution should be administered for ongoing losses through diarrhea.
• No unnecessary medications should be administered.

King CK, Glass R, Bresee JS, Duggan C, Centers for Disease Control and Prevention: Managing acute gastroenteritis among children: oral rehydration, maintenance, and nutritional therapy, MMWR Recomm Rep 52:1–16, 2003.

224. What is the most common treatment for travelers’ diarrhea in children?
Travelers’ diarrhea most commonly is caused by enterotoxigenic E. coli (ETEC). If symptoms develop, empiric therapy is indicated. Effective antibiotic regimens include azithromycin (for all children), trimethoprim-sulfamethoxazole (for children >2 years), a quinolone (e.g., ciprofloxacin, norfloxacin) or rifaximin (for children 12 years). The use of an anti-motility agent (e.g., loperamide) is also considered beneficial.
225. Which patients are particularly susceptible to giardiasis?
Those with cystic fibrosis, chronic pancreatitis, achlorhydria, agammaglobulinemia, and hypogammaglobulinemia

226. What common conditions are associated with an increased risk for celiac disease?
• Type 1 diabetes
• Autoimmune thyroiditis
• Down syndrome
• Turner syndrome
• William syndrome
• Selective IgA deficiency
• First-degree relatives with celiac disease

Rodrigues AF, Jenkins HR: Investigation and management of coeliac disease, Arch Dis Child 93:251–254, 2008.

227. What is the management for massive upper GI bleeding in children?
This type of hemorrhage is a life-threatening emergency, and initial therapy precedes the specific diagnostic evaluation. Management includes the following:
• Brief history and character of bleeding, previous episodes, and bleeding disorders
• Studies (CBC, liver function tests, coagulation profile, crossmatch to blood bank)
• Nasogastric tube insertion
• Full history and physical examination
• Transfusion and intravascular support
• Determination of probable etiology
Peptic ulcer disease: Diagnostic endoscopy; therapeutic endoscopy; H2 blockers, antacids, sucralfate
If no resolution: Surgical repair of ulcer, partial resection
Variceal bleeding: Diagnostic endoscopy; therapeutic endoscopy; vasopressin, octreotide
If no resolution: Sengstaken-Blakemore tube, emergency portosystemic shunt, esophageal devascularization
Mallory-Weiss tear
Superficial VASCULAR anomaly: Endoscopic ablation

Chawla S, Seth D, Mahajan P et al: Upper gastrointestinal bleeding in children, Clin Pediatr 46:16–21, 2007.

228. Why is the buffering of gastric acid important for controlling upper GI bleeding?
• Acid is ulcerogenic and can cause and propagate erosions.
• Coagulation is better in a neutral or alkaline environment than in an acidic one.
• Platelet plugs are disrupted by gastric pepsins, but these pepsins function less well in a neutral or alkaline environment.

Mezoff AG, Preud’homme DL: How serious is that GI bleed? Contemp Pediatr 11:60–92, 1994.

229. What are the most common complications of fundoplication?
Dysphagia can result if the wrap is too tight. Small bowel obstruction and paraesophageal hernia can be postsurgical complications. Gas-bloat syndrome, characterized by persistent gagging, retching, nausea, and abdominal distention, is reported. Post-fundoplication dumping syndrome is also a known complication.

230. How does the presentation of achalasia differ with age?
Clinical presentation depends on age of the patient, duration of the disease process, and is often insidious. Infants and toddlers may present with choking, cough, recurrent chest infections, feeding aversion, and FTT. Older children usually present with vomiting, dysphagia, weight loss, respiratory symptoms, and slowed rate of eating. Dysphagia may be related to solids initially, but will progress to affect liquids as well.
231. Can hepatitis C virus (HCV) be acquired vertically at birth?
Yes. The mechanisms underlying vertical transmission of HCV are poorly understood. Intrauterine transmission during pregnancy and infection at the time of delivery are both possible. Estimates on the rate of vertical transmission of HCV vary significantly. Overall, it appears that the risk is approximately 2%, with higher rates in certain subgroups, such as women who are co-infected with human immunodeficiency virus (HIV). The mode of delivery does not appear to be associated with the risk of vertical transmission of HCV. Progression to chronicity occurs in most infants with vertically acquired HCV. Liver disease usually is mild throughout childhood.
232. Is a decreased ceruloplasmin pathognomonic to Wilson disease?
No. Ceruloplasmin is the major copper-carrying protein in the blood. While low levels are part of the diagnosis of Wilson disease, decreased ceruloplasmin levels can be also be seen in malnutrition, protein-losing enteropathy, nephrotic syndrome, hepatic insufficiency, neonates, Menkes syndrome, and patients who are heterozygote for Wilson disease.
233. An infant with cholestasis, triangular facies, and a pulmonic stenosis murmur is likely to have what syndrome?
Alagille syndrome (arteriohepatic dysplasia). Also called syndromic bile duct paucity, this condition consists of a constellation of conjugated hyperbilirubinemia and cholestasis; typical triangular facies; cardiac lesions of pulmonic stenosis; peripheral pulmonic stenosis; or, occasionally, more significant lesions; butterfly vertebrae; and eye findings of posterior embryotoxon and Axenfeld anomaly or iris processes. The patient may have extreme cholestasis with pruritus and marked hypercholesterolemia. Although some patients have developmental delay, most develop appropriately. The usual mode of inheritance of Alagille syndrome is autosomal dominant.
234. What are the common side effects of the immunomodulators?
• Methotrexate: Nausea, leukopenia, liver toxicity, decreased appetite, increased infection risk, reaction at injection site
• 6-mercaptopurine/ azathioprine: Nausea, leukopenia, liver toxicity, pancreatitis, increased infection risk, increased risk of malignancy
235. What is the difference between Infliximab and adalimumab?
Infliximab is a chimeric monocolonal antibody that uses a variable murine region and is administered via regular intravenous infusions. Adalimumab is a fully humanized antibody that is given via regular subcutaneous injections. Infliximab is considered more antigenic than adalimumab because of its murine region, and patients have a greater likelihood of developing antibodies to the medication, which decreases efficacy.
236. What is the mechanism of enteral nutrition therapy for IBD?
Enteral nutrition therapy, also known as tube feeding, is initiated with the use of a pediatric formula to deliver the majority of calorie needs. The type of formula and percentage of total calories delivered via formula vary by institution and region. The proposed mechanism of action is conjectured but hypotheses include elimination of dietary antigen uptake, nutritional repletion, correction of intestinal permeability, and alteration of gut microflora.

Alhagamhmad MH, Day AS, Lemberg DA, et al: An update on the role of nutritional therapy in the management of Crohn’s disease, J Gastroenterol 47:872–882, 2012.

237. What is the postoperative prognosis for Crohn disease and ulcerative colitis? Patients with ulcerative colitis who undergo colectomy with ileal pouch anal anastomosis should not expect recurrence of disease. However, pouchitis, or inflammation of the ileal pouch that acts

as a neorectum, is not uncommon. Alternatively, surgical resection is not curative for Crohn disease. The overall rate of clinical recurrence is estimated to be 50% 5 years after the initial resection.
Medical therapy is often required after resection to decrease the risk for recurrence.
238. When and how should solids be introduced into an infant’s diet?
Supplemental foods can be added to an infant’s diet starting at 4 to 6 months when they can sit with support; have good head and neck control; show readiness for various textures by putting thing in their mouths; have doubled their birth weight; and need supplementation to their liquid diet. It is also the time of initial tooth eruption. Introduction of solid foods earlier to this time may interfere with the ability to take adequate calories. In addition, infants may not have the
coordination/skills to safely swallow solids, which could cause aspiration; however delaying solids past 6 months may lead to decreased growth because they may not get enough calories from breast milk or formula alone. First foods should be single-grain infant cereals followed by
single-ingredient pureed foods introduced one at a time every few days. It is no longer recommended that highly allergenic foods such as eggs, peanuts, tree nuts, fish, and shellfish be delayed because it has not been shown to help prevent allergy. It is controversial whether early introduction of solid foods might lessen the likelihood of atopic diseases. Hard or round foods such as nuts, grapes, raw carrots, candy, etc. should be avoided because they pose a choking hazard.
239. What are risk factors for low vitamin D levels?
• Premature infants
• Vitamin D deficiency in breastfeeding mother
• Prolonged exclusive breastfeeding
• Poor diet
• Lack of sunlight exposure/sunscreen
• Dark skin pigmentation
• Malabsorption (cystic fibrosis, IBD, celiac disease)
• Obesity
• Medications (anticonvulsants, glucocorticoids)
• Winter and spring months
240. What comorbidities are associated with obesity?
Endocrine: impaired glucose tolerance, type 2 diabetes mellitus, polycystic ovarian syndrome/ hyperandrogenism, hypothyroidism, metabolic syndrome
Cardiovascular: hypertension, dyslipidemia, early cardiovascular disease
Pulmonary: obstructive sleep apnea, asthma
GI/Hepatic/Nutrition: vitamin D deficiency, nonalcoholic fatty liver disease (NAFLD), cholelithiasis Musculoskeletal: slipped capital femoral epiphysis (SCFE), tibia vara (Blount disease), fractures Neurologic: idiopathic intracranial hypertension (pseudotumor cerebri)
Dermatologic: acne, acanthosis nigricans, skin infections such as intertrigo, furunculosis, hidradenitis suppurativa
Psychosocial: depression, anxiety, distorted body image, poor self-esteem

241. How do causes of intestinal obstruction vary by age?
Infant and young child:
• Pyloric stenosis
• Intussusception
• Inguinal hernia
• Appendicitis
• Malrotation
• Intestinal duplication
• Intestinal atresia or stenosis
• Omphalomesenteric remnants
• Intraluminal web
• Hirschsprung disease
• Adhesions
Older child:
• Appendicitis (perforated)

• Intussusception (lead point)
• Adhesions
• Malrotation
• Inguinal hernia
• Omphalomesenteric remnants
• IBD

Caty MG, Azizhan RG: Acute surgical conditions of the abdomen, Pediatr Ann 23:192–194, 199–201, 1994.

242. When should a Kasai procedure be performed? The Kasai procedure should be done as soon as possible. Earlier operation results in a dramatically improved outcome. Patients operated on when they are younger than 70 days have an increased likelihood of a successful procedure, although exceptions at both ends of this spectrum are common. Although some surgeons now suggest that infants diagnosed late in the course of
disease should have a primary liver transplantation rather than a hepatoportoenterostomy because sufficient liver injury has occurred to make the Kasai procedure unlikely to be successful, there are some data to suggest that late portoenterostomy might be beneficial and result in medium-term survival with native liver in up to one-third of patients.

Davenport M, Puricelli V, Farrant P, et al: The outcome of the older (> or 100 days) infant with biliary atresia,
J Pediatr Surg 39:575–581, 2004.

243. What conditions may mimic appendicitis?
• Gastroenteritis
• Ruptured ovarian follicle and ovarian torsion
• Mesenteric adenitis
• IBD
• Constipation
• Henoch-Sch€onlein purpura
• Pelvic inflammatory disease
• Primary peritonitis
• Pyelonephritis
• Perforated peptic ulcer
• Right lower lobe pneumonia
• Pancreatitis

Caty MG, Azizhan RG: Acute surgical conditions of the abdomen, Pediatr Ann 23:192–194, 199–201, 1994.